Category Archives: IAS 2011 Gs Model paper I II

IAS 2011 Civil Services Part 2 General Study Answer Key

CSAT June 2011 answer key for General Studies Paper II: Shanker IAS academy has published the answer key of IAS Civil Services 12-6-2011answer key for paper 2 of GS. Besides detailed solution and answer key, Institute has also released the answer key for all series.

Download Answer sheet Paper 2 GS IAS Civil Services exam 

IAS Prelims CSAT 2011, Model Question Paper (Solved) Set – 1

1. Amar, Akbar, Anthony are friends, being looked after a matron Farah,Amar weighs 50% more than Akbar and Anthony weighs 25% less than Amar.Farah weighs a third of the combined weight of the three boys.All four together weight 232Kg. The correct arrangement of the person in the ascending order of their weights, is:
(a) Anthony, Akbar, Farah, Amar
(b) Anthony, Akbar, Amar, Farah
(c) Akbar, Anthony, Amar, Farah
(d) Akbar, Anthony, Farah, Amar

Ans. (d) 

2. If x=-2, then x3-x2-x-1 is equal to
(a) 1
(b) -3
(c) -11
(d) -15

Ans. (c)

3.
 Examine the following relationships among members of a family of six persons A, B, C, D, E and F.
1. The number of males equals that of females
2. A and E are sons of F
3. D is the mother of two, one boy and one girl
4. B is the son of A
5. There is only one married couple in the family at present
Which one of the following inferences can be drawn from the above?
A, B and C are all females
A is husband of D
E and F are children of D
D is the grand daughter of F

Ans. (b)

4. The missing fraction in the series given below is
4/ 9, 9 / 20, …. 39 / 86
(a) 17 / 40
(b) 19 / 42
(c) 20 / 45
(d) 29 / 53

Ans: (b)

5. In an examination, every candidate took physics or mathematics or both 65.8% took physics and 59.2% took mathematics. The total number of candidates was 2000. How many candidates took both physics and mathematics ?
(a) 750
(b) 500
(c) 250
(d) 125

Ans: (b)

6. A bag contains 20 balls. 8 balls are green, 7 are white and 5 are red. What is the minimum number of balls that must be picked up from the bag blind-folded (without replacing &.y of it) to be assured of picking atleast one ball of each colour ?
(a) 4
(b) 7
(c) 11
(d) 16

Ans: (d)

Comprehension

Instructions (Q 7-17): Read the following passage carefully and answer the questions given below it.
”If we are so contemptibly selfish that we can’t radiate a little happiness and pass on a bit of honest appreciation without trying to get something out of the other person in return-if our souls are no bigger than sour crab aples, we shall meet with the failure we so richly deserve. Oh yes, I did want something out of that chap. I wanted something priceless. And I got it. I got the feeling that I had done something for him without his being able to do anything whatever in return for me. That is a feeling that flows and sings in your memory lung after incident is past.
There is a one all-important law of human conduct. If we obey that law, we shall almost never get into trouble. In fact, that law, if obeyed, will bring us countless friends and constant happiness. But the very instant we break the law, we shall get into endless trouble. The law is the: Always make the other person feel important.John Dewey, as we have already noted, said that the desire to be important is the deepest urge in human nature; and William James said, “The deepest principle in human nature is the craving to be appreciated.” As I have already pointed out, it is this urge that differentiates us from the animals. It is this urge that has been responsible for civilization itself.
Philosphers have been speculating on the rules of human relationships for thousands of years, and out of all that speculation, there has evolved only one important precept. It is not new. It is as old history. Zoroaster taught it to his followers in Persia twenty-five hundred years ago. Confucius preached it in China twenty-four centuries ago. Lao-tse, the founder of Taoism, taught it to his disciples in the Valley of the Han. Buddha preached it on the bank of the Holy Ganges five hundred years before Christ. The sacred books of Hinduism taught it a thousand years before that. Jesus taught it among the stony hills of Judea nineteen centuries ago. Jesus summed it up in one thought-probably the most important rule in the World: “Do unto others as you would have others do unto you.”
You want the approval of those with whom you come in contact. You want recognition of your true worth. You want a feeling that you are important in your little world. You don’t want to listen to cheap, insincere flattery, but you do crave sincere appreciation. You want your friends and associates to be, as Charles Schwab put it, “hearty in their approbation and lavish in their praise.” All of us want that.”

7. According to the author, when do human being become extremely selfish?
(a) By desiring to be appreciated
(b) By the craving for being important
(c) By not giving others even just the feeling of appreciation without extracting somethings in return
(d) By receiving past memories

Ans: (c)

8. What is ‘Taoism’?
(a) A belief of living a high and happy life
(b) A belief of living a simple honest life and non-interference with the course of natural events
(c) A belief of living a life with ethical and spiritual principles
(d) None of the above

Ans: (b)

9. In the third paragraph of the above passage, with all the examples, what teaching does author indicate?
(a) Teching Taoism
(b) Philosphical teaching
(c) The rules prevailed in the ancient time

Ans: (c)

10. With which of the following sentences, you car replace “Do unto oyhers as you would have others do unto you”?
(a) Do for others as you would have others do for you
(b) Do to others as you would have others do to you
(c) Do before others as you would have others do before you
(d) Do after others as you would have others do after you

Ans: (b)

11. What according to the author does differentiate human beings from the animals?
1. Desire to be important
2. Desire to be praised
3. Urge to be happy constantly
4. Urge to end all trouble
(a) 1 Only
(b) 1 & 2
(c) 1, 2 & 3
(d) 1, 2, 3 & 4

Ans: (b)

12. Which of the following expresses something very bad and deserving no respect?
(a) Contemptible
(b) Radiate
(c) Approbation
(d) None of the above

Ans: (a)

13. In the first paragraph, what does the word, ‘radiate’ mean?
(a) To spread out from a central point
(b) To emit or be emitted as radiation
(c) To show an emotion or quality to a great degree
(d) To manifest in a glowing manner

Ans: (c)

14. “Always make the other person feel important” By this line what is highlest?
(a) Human emotion
(b) Human nature
(c) Human conduct
(d) All of the above

Ans: (c)

15. What is the anitonym of ‘lavish’?
(a) Lifeless
(b) Beautiful
(c) None of the above

Ans: (b)

16. Suggest a suitable title for the passage from the options below?
(a) Ways to win peoples influence
(b) Ways to make friends
(c) An all important law of human conduct
(d) Make others feel important

Ans: (a)

17. A man purchases two clocks A and B at a total cost of Rs. 650. He sells A with 20% profit and B at a loss of 25% and gets the same selling price for both the clocks. What are the purchasing price of A and B respectively
(a) Rs. 225 ; Rs. 425
(b) Rs. 250 ; Rs. 400
(c) Rs. 275 ; Rs. 375
(d) Rs. 300 ; Rs. 350

Ans: (b)

18. If 15 pumps of equal capacity can fill a tank in 7 days, then how many extra pumps will be required to fill the tank in 5 days’?
(a) 6
(b) 7
(c) 14
(d) 21

Ans: (a)

19. Out of three annual examinations, each with a total of 500 marks, a student secured average marks of 45% and 55% in the first and second annual examinations. To have an overall average of 60%, how many marks does the student need to secure in the third annual examination ?
(a) 450
(b) 400
(c) 350
(d) 300

Ans: (b)

20. Which one of the following satisfied the relationship
Dda : aDD :: Rrb: ?
(a) D D A
(b) R R R
(c) b R R
(d) B B R

Ans: (c)

21. A, B, C, D, E and F not necessarily in that order are sitting in six chairs regularly placed around a round table. It is observed that A is between D and F C is opposite D D and E are not on neighbouring chairs Which one of the following must be true ?
(a) A is opposite B
(b) D is opposite E
(c) C and B are neighbours
(d) B and E are neighbours

Ans: (d)

22.
 In a class there are 18 boys who are over 160 cm tall. If these boys constitute three-fourths of the boys and the total number of boys is two-third of the number of students in the class, then what is the number of girls in the class ?
(a) 6
(b) 12
(c) 18
(d) 24

Ans: (b)

23. Read the following chain of connected statements carefully and find out which category they belong to out of Fact (F), Argument (A), Prejudice (P), Judgments (J) and Opinion?
a) the condition of women seems to have become better than of men these days in India.
(b) because wherever we see, women are seen having succeeded in most of the areas
(c) we have Pratibha Devi Singh Patil as a woman president
(d) and also many successful women Politicians and Chief ministers like Sonia Gandhi, Mamta Banerjee, Ms. Jayalalita, Ms. Mayavati, Sheela Dixit etc.
(e) it proves that women are not less capable than men
(f) Probably, that day is not far when reservation will be required for men.

(a) (J), (F), (F), (J) and (O)
(b) (O), (F), (F) ,(A),(J) and (P)
(c) (P), (A), (F), (F), (J) and (O)
(d) (O), (A), (F), (F)  and (O)

Ans: (d)

24. Read the following statements carefully. These are the statements of a continuous series but they have been kept in disarranged order. You have to arrange them in the right order in such a way that the passage gets completed while reading them together.

(1) because it was limited to middle class leaders.
(2) Gandhiji had amazing communication skills
(3) This was the reason that when communication mediums like T.V. Radio etc. were not available
(4) Indian freedom struggle was incomplete before the arrival of Mahatma Gandhi
(5) and mass participation of people was not there in it
(6) he made full use of the power inherent in symbols like Dandi Mission, Charkha, Swadeshi etc.
(7) still, he succeeded in establishing Communication and farming consensus throughout the whole country

(a) (2), (3),(7), (6), (4),(1) and (5)
(b) (4), (2), (3),(7), (6), (1) and (5)
(c) (4), (1), (5), (2), (7), (3) and (7)
(d) (2), (3), (7), (6), (5), (1) and (4)

Ans: (c)

25. Assume that a much awaited movie of a renowned film director was going to release. The Subject of the Movie was also related to society and quite relevant which attracted audience to a large extent. Still, the movie was a big flop. What can be the reasons of it in your opinion? Choose four most important reasons out of the following options-

(1) The story of the movie must be lacking the ability of making audience stick to it.
(2) The songs of the movie must deviated from the story or be boring.
(3) The movie must have been too long.
(4) the movie must have been based on the problem of such social class which would not have sufficient cognitive ability to understand or sufficient purchasing power to watch the movie
(5) The movie must have been relaased during the time of Cricket world cup or any such entertaining event
(6) Any big star must have been casted as Hero in the movie

(a) (1), (4), (5) and (3)
(b) (6), (4), (2) and (1)
(c) (1), (6), (5) and (4)
(d) (4),  (1), (6) and (2)

Ans: (a)

26. (65.5 x 15.5) x 100 =?
(a) 108125
(b) 102615
(c) 101525
(d) 105125    

Ans: (c)

  
27. If7x + 8y= 53 and 5x + 12y=63, then what is the value of (y-x)?
(a)-2
(b) 2
(c) 1
(d)-1    

Ans: (c)             

28. A man walks at the speed of 5 km/hr and runs at the speed of 1 0 kmIhr. How much time will the man require to cover the distance of28 km, if he covers half(first 14 km) his journey walking and half his journey running?

(a) 8.4 hrs
(b) 5 hrs
(c) 4.2 hrs
(d) None

Ans: (c)

29. If the positions of the first and the third digits in each of the above numbers are interchanged, which of the following will be the third highest number?

(a) 851
(b) 743
(c) 624
(d) 319

Ans: (c)

30. How many such pairs of letters are there in the word SOUTHERN each of which has as many letters between them in the word as in the English alphabet?
(a) None
(b) One
(c) Two
(d) Three

Ans: (d)

31. The positions of the first and the fifth digits in the number 84329517 are interchanged. Similarly, the positions of the second and the sixth digits are interchanged; and so on. ‘Which of the following will be the second element from the right end after the rearrangement?
(a) 3
(b) 4
(c) 2
(d) None of these

Ans: (a)

32. In a certain code TOLD is written as 5132 and DEAR is written as 2648. How is LATE written in that code?

(a) 3652
(b) 3246
(c) 3856
(d) 3456

Ans: (d)

33.
 If in a certain code SAND is VDOG and BIRD is ELUG, then what is the code for LOVE
(a) PRYG
(b) ORTG
(c) NPUH
(d) ORYH

Ans: (d)
34. In a family, a couple has a son and a daughter. The age of the father is three times that of his daughter and the age of the son is half of his mother. The wife is nine years younger to her husband and the brother is seven years older than his sister. What is the age of the mother’?
(a) 40 years
(b) 45 years
(c) 50 years
(d) 60 years

Ans: (d)

35. A car travels the first one-third of a certain distance with a speed of a certain distance with a speed of 10km/hrs. The next one-third of distance with a speed of 20 km/hrs. and the last one-third distance with a speed of the car for the whole journey  is :
(a) 18km/hrs.
(b) 24km/hrs.
(c) 30km/hrs.
(d) 36km/hrs.

Ans. (a)
36. Total time taken by a person in going to a place by walking and returning on cycle is 5 hours 45 minutes. He would have gained 2 hours by cycling both ways. The time taken by him to walk both ways, is
(a) 6 hours 45 minutes
(b) 7 hours 45 minutes
(c) 8 hours 15 minutes
(d) 8 hours 30 minutes

Ans. (b)

37. In a question of a test paper, there are five items each under List-A and List-B. The examinees are required to match each item under List-B. Further, it is given that
(i) no examinee has given the correct answer.
(ii) answer of no two examinee are identical which is the maximum number of examinees who took this test ?
(a) 24
(b) 26
(c) 119
(d) 129

Ans : (d)

38. Left pan of faulty weighs 100gram more than is right pan. A shopkeeper keeps the weight measure in the left pan while buying goods but keeps it in the right pan while selling his goods.He uses only 1kg weight measure. If he sells his goods at the listed cost price, What is his gain?
(a) 200/11%
(b) 100/11%
(c) 1000/9%
(d) 200/9%

Ans : (a)

39.
 Assume that you are busy ina meeting with your senior officers. During the meeting, your immediate boss is asked about the progress of some work whose answer he doesn’t have. He was supposed to do that work but he had forgotten it due to some reason. He puts the whole blame on you in order to avoid his embarrassment and scolds you in front of all in the meeting. What will be your reaction in such situation? Choose the best option:
(a) Will remain quiet at that point of time but will express your apposition later when alone.
(b) Will Express your apposition openly in front of all
(c) Will give your explanation politely but firmly in front of all but will not blame him directly.
(d) Will leave the meeting quietly without saying anything for expressing your anger on false blame.

Ans: (c)

40. How many such numerals are there in the number 2576489, which will remain at the same position when arranged in ascending order from left to right?
(a) One
(b) Two
(c) Three
(d) More than three

Ans: (d)

41. Four of the following five are alike in a certain way and so form a group. Which is the one that does not belong to that group?
(a) Coriander
(b) Mustard
(c) Cumin
(d) Clove    

Ans: (d)

42. ‘Visual’ is related to ‘Light’ in tbe same way as ‘Audio’ is related to
(a) Voice
(b) Sound
(c) Noise
(d) None

Ans: (b)

43. ‘Radish’ is related to ‘Root’ in the same way as ‘Brinjal’ is related to
(a) Fruit
(b) Stem
(c) Flower
(d) Root

Ans: (a)

Instructions for the following Question:  Read the following information to answer the question given below.
The following conditions have been decided for the selection of Research Assistants. The candidate must:

(i) Be less than 35 years of age.
(ii) Be graduate in Psychology, Education or Commerce or Postgraduate in any other subject.
(iii) Secure 50% marks in entrance examination.
(iv) Secure A or O grade in Interview.
(v) Have passed a certificate course in Methodology Research.
(vi) Have recommendation of any one lecturer. If a candidate fulfills all the conditions, except

(a) (i), but if he is registered for Ph.D., his case is to be kept in waiting list.
(b) (iii), but if he has secured 45% marks in entrance examination and O grade in interview, his case is to be referred to the Dean.
(c) (v), but if he has qualified certificate course in Statistics, his case is to be referred to the Head Of Department.
You have to decide in each of the following cases that what is the possibility of selection of the candidate? If the information is not sufficient to take any decision then your answer will be ‘data inadequate’. This information has been given to you as on 15.12.1996.

44. Jaya was born on 11.10.1962. She is a Postgraduate in Statistics and ahs qualified certificate course in Methodology Research. She is registered for Ph.D and has secured 52% marks in entrance examination. She has got O Grade in interview.
(a) will not be selected.
(b) Will be selected.
(c) Will be kept in waiting list.
(d) Data inadequate

Ans: (d)

45. What is maximum number of pieces of 5 cm x 5 cm x 10 cm cake that can be cut from a big cake of 5 cm x30 cm x 30 cm size?
(a) 10
(b) 15
(c) 18
(d) 30

Ans: (c)

46. A rectangular water tank measures 15 m x 6 m at top and is 10 m deep. It is full of water. If water is drawn out lowering the level by 1 meter, how much of water has been drawn out ?
(a) 90,000 litres
(b) 45,000 litres
(c) 4,500 litres
(d) 900 litres

Ans: (a)

47. An accurate clock shows 8 O’ clock in the morning. Through how many degrees will the hour hand rotate when the dock shows 2 O’clock in the afternoon ?
(a) 150o
(b) 144o
(c) 168°       
(d) 180°

Ans: (d)

48. The monthly income of Komal and Asha are in the ratio of 4 : 3. Their monthly expenses are in the ratio of 3: 2. However, both save Rs. 600/- per month. What is their total monthly income?
(a) Rs. 8,400 /-
(b) Rs. 5,600 /-
(c) Rs. 4,200 /-
(d) Rs. 2,800 /-

Ans: (c)

49. Examine the following statements
1. All members of Mohan’s family are
2. Some members of Mohan’s family are not employed.
3. Some employed persons are not honest.
4. Some honest persons are not employed.

Which one of the following inferences can be drawn from the above statements ?

(a) All members of Mohan’s family are employed
(b) The employed members of Mohan’s family are honest
(c) The honest members of Mohan’s family are not employed
(d) The employed members of Mohan’s family are not honest

Ans: (b)

Instructions (Q. 50-51): Two question given below have a statement each followed by two assumptions which have been represented by serial No. (I) and (II). Any assumed or adopted belief is called an Assumption. Keeping the given statement and the given assumption in mind, you have to decide which of the assumptions is/are inherent in the statement? Give your answer according to the following options:
Answer (a), if only Assumption (i) is inherent.
Answer (b), if only Assumption (II) is inherent.
Answer (c), if neither Assumption  (I) is  (II) inherent.
Answer(d), if both Assumption (I) and (II) are inherent.

50. Statement: Female Foeticide cannot be eradicated from Indian society unless we change the Indian peoples’ mindset.
Assumptions: I. Existence of Female Foeticide in Indian society is not bad.
II. Changing people’s mindset is not impossible.

Ans: (b)

51. Statement: S.B.I., which is the largest bank of India, has decided to reduce its work force by 25 percent so that it’s branches may work efficiently.
Assumptions:
I. Bank can perform better in all its activities after the reduction in work force.
II. The surplus employees may be asked to accept early retirement scheme.

Ans: (a)

52. There are 50 students admitted to a nursery class. Some students can speak only English and some can speak only Hindi. 10 students can speak both English and Hindi. If the number of students who can speak English is 21, then how many students can speak only Hindi, how many can speak only Hindi and how many can speak only English?
(a) 21, 11 and 29 respectively       (b) 28, 18 and 22 respectively
(c) 37, 27 and 13 respectively       (d) 39, 29 and 11 respectively

Ans: (d)
53. An accurate clock shows the time as 3.00. After hour hand has move 135°, the time would be
(a) 7.30
(b) 6.30
(c) 8.00
(d) 9.30

Ans: (a)
54. One local and another express train were proceeding in the same direction on parallel tracks at 29km/hourand65 km/hour respectively. The driver of the former notices that it look exactly 16 seconds for the faster train to pass by him. What is the length of the faster train?
(a) 60 m
(b) 120 m
(c) 160 m
(d) 240 m

Ans: (c)

55.
 Which one of the following has a greater perimeter than the rest ?
(a) A square with an area of 36 sq. cm.
(b) An equilateral triangle with a side of 9 cm.
(c) A rectangle with 10 cm as length and 40 sq. cm. as area
(d) A circle with a radius of 4 cm.

Ans: (c)

56. A club has 1,08 members. Two-thirds of them are men and the rest are women. All members are married except for 9 women members. How many married women are there in the club?
(a) 20
(b) 24
(c) 27
(d) 30

Ans: (c)

57. A rectangular piece of iron sheet measuring 50 cm by 100 cm is rolled into a cylinder of height 50 cm. If the cost of painting the cylinder is Rs. 50/- per square meter, then what will be the cost of painting the outer surface of the cylinder ?
(a) Rs.25.00
(b) Rs.37.50
(c) Rs.75.00
(d) Rs.87.50

Ans: (a)

58. An equilateral triangular plate is to be cut in to n number of identical small equilateral triangular plates. Which of the following can be possible value of n?
a) 196
b) 216
c) 256
d) 296

Ans : (c)

Comprehension

Instructions (Q 59-69): Read the following passage carefully and answer the questions given below it.

“I discovered, from the analysis of over 25,000 people,that men who succeed in an outstanding way, seldom do so before the age of forty, and more often they do not strike their real pace until they are well beyond the age of fifty, This fact was so astounding that it prompted me to go in to the study of its cause most carefully, carrying the investigation over a period of more than twelve years.
This study disclosed the fact that the major reason why the majority of men who succeed do not begin to do so before the age of fourty to fifty, is their tendency to DISSIPATE their energies through over indulgence in physical expression of the emotion of sex. The majority of men never learn that the urge of sex has other possibilities, which for transcend in importance, that of mere physical expression. The majority of those who make this discovery, do so after having wasted many years at a period when the sex energy is at its height, prior to the age of fourty-five to fifty.This usually is followed by noteworthy achievement.
The lives of many men up to, and sometimes well past the age of fourty, reflect a continued dissipation of energies, which could have been more profitably turned into better channels.Their finer and more powerful emotions are sown wildly to the four winds.Out of this habit of the male, grew the term, “sowing his wild oats.”
The desire for sexual expression is by far the strongest and most impelling of all the human emotions, and for this very reason this desire, when harnessed and transmutted in to action, other than that of physical expression, many raise one to the status of a genius.
One of America’s most able business men frankly admitted that his attractive secretary was responsible for most of the plans he created.He admitted that her presence lifted him to heights of creative imagination, such as he could experience, under no other stimulus.
One of the most successful men in America owes most of his success to the influence of a very charming young woman, who has served as his source of inspiration for more than twelve years.
Everyone knows the man to whom this reference is made, but not everyone knows the REAL SOURCE of his achievements.”

59. What took the author to carry an investigation for more than twelve years?
1. To find the reason why men seldom succeed before fourty
2. To find out if actually men do not strike their real pace until they are well beyond the age of fifty
3. To find the cause of success in a outstanding way
4. To find the reason behind a real pace in men
(a) 1 Only        (b) 2 Only
(c) 1 & 2          (d) 3 & 4

Ans: (c)

60. What is the synonym of ‘austounding’?
(a) Interesting
(b) Boring
(c) Surprising
(d) Extremely Surprising

Ans: (d)

61. According to the author, what is the reason for the majority of men’s failure in achieving success?
(a) Their a tendency to dissipate time
(b) Their tendency to waste a good amount of energy through over indulgence in physical expression of the emotion of sex
(c) Their mere involvement in physical expression
(d) All the above

Ans: (b)

62. What finally puts an end to dissipale men’s energy?
1. Realisation
2. An important achievement
3. When men reach the age of fourty-five
4. When sex energy is at its height
(a) 1 Only         (b) 2 Only        (c) 1 & 2        (d) All of the above

Ans: (c)

63. “Sowing his wild oats” is an out comes of which of the following?
(a) The continuation of dissipation of energies
(b) Using energy in a better way
(c) Finer practice of the more powerful energies
(d) All of the above

Ans: (a)

64. How can a genius status be achieved?
(a) by stopping the sexual urge
(b) by the attaining the age of fourty-five
(c) by harnessing and transmuting sexual desire into action
(d) None of the above

Ans: (c)

65. From the above passage which message is given?
(a) Behind every successful man, there in a woman
(b) Men reach success after the age of fourty
(c) Success can be achieved by transmuting sexual sexual desire into action
(d) Success starts at the pause of sexual desire

Ans: (c)

66. By examples cited above, what does the author indicate?
(a) One can prepare finer plans in the presence of a woman
(b) One can do work better when the physical stimulus is transformed into action by the woman’s influence
(c) Woman can do better work
(d) Men can do better work

Ans: (b) 

67.
 Which of the following terms explain to waste some things by not using it in a sensible way?
(a) Astound
(b) sow
(c) dissipate
(d) None of these

Ans: (c)

68.
 Consider  the following statements:
1. The urge for sex could have other possibilities
2. woman can influence in transforming mens physical emotions in to better work
3. men experience success only after the age of fourty
4.  An noteworthy achievement after the age of fourty can put an end to dissipate men’s energy by the physical involvement.
Which of the above statement/ statements is/are correct?
(a)1 & 2        (b) 1, 2, 3        (c) 1, 2 & 4        (d) 3 Only

Ans: (c)

69. There are two numbers such that the sum of twice the first number and thrice.the second number is 100 and the sum of thrice the first number and twice the second number is 120. Which is the larger number?
(a) 32
(b) 12
(c) 14
(d)35    

Ans: (a)

70. When-all the students in a school are made to stand in rows of 54, 30 such rows are formed. If the students are made to stand in rows of 45, how many such rows will be formed?
(a) 25
(b) 42
(c) 32
(d) None of these

Ans: (c)

71. The ratio of the present ages of Anju and Sandhya is 13: 17. Four years ago the ratio of their ages was II: 15. What will be the ratio of their ages six years hence?
(a) 3:4
(b) 7:8
(c) 5:4
(d) None of these

Ans: (d)

72. The average age of80 boys in a class is 15. The average age of a group of 15 boys in the class is 16 and the average age of another 25 boys in the class is 14. What is the average age of the remaining boys in the class?
(a) 15.25
(b) 14
(c) 14.75
(d) Cannot be determined

Ans: (a)

Directions (Q. 73-77): Rearrange the following six sentences (A), (B), (C), (D), (E) and (F) in the proper sequence to form a meaningful paragraph; then answer the questions given below.
(A) With all the bid information being available and tracked online, corruption has considerably reduced.
(B) Today, most, ie over ninety-five per cent, households, in the city enjoy. broadband connection.
(C) All city contracts are now bid for online.
(D) Over twenty years ago the city government, Central Government and the private sector made a concerted effort to shift the economy to include IT.
(E) As our cities do expand and become more complex, such a system will make governance more
manageable.
(F)’ This level of connectedness has changed not only the city’s economy but also how it is governed and
how business is conducted.

73. Which of the following should be the FIRST sentence after rearrangement?
(a) B
(b) C
(c) D
(d) E    

Ans: (d)

74. Which of the following should be the SECOND sentence after rearrangement?
(a) A
(b) B
(c) C
(d) D    

Ans: (a)

75. Which of the following should be the THIRD sentence after rearrangement?
(a)A
(b)C
(c)D
(d)E    

Ans: (c)

76. Which of the following should be the FIITH sentence after rearrangement?
(a)A
(b)E
(c)C
(d)B

Ans: (b)

77. Which of the following should be the LAST (SIXTH) sentence after rearrangement? 
(a)B
(b)C
(c)D
(d)E    

Ans: (a)

Directions (78-82): Study the following information carefully and answer the given questions: A word and number arrangement machine when given an input line of words and numbers rearranges them following a particular rule in each step. The following is an illustration of input and rearrangement.
Input     gone 93 over 46 84 now for 31
Step I     31 gone 93 over 46 84 now for
Step II     31 over gone 93 46 84 now for
Step Ill    31 over 46 gone 93 84 now for
Step IV     31 over 46 now gone 93 84 for
Step V     31 over 46 now 84 gone 93 for
and Step V is the last step of the rearrangement of the above input. As per. the rules followed in the above steps, find out in each of the following questions the appropriate step for the given input.

78. Step III of an Input: 15 window 2993 86 sail tower buy Which of the following will be step VI?
(a) 15 window 29 tower 86 sail 93 buy
(b) 15 window 29 tower 86 93 sail buy
(c) 15 window 29 tower 93 86 sail buy
(d)There will be no such step

Ans: (a)

79. Input: station hurry 39 67 all men 86 59. How many steps will be required to complete the rearrangement?
(a) Four
 (b) Five
(c) Six
(d) Three    

Ans: (b)

80. Step II of an input is : 49 zone car battery 56 87 71 down Which of the following is definitely the input?
(a) car 49 battery zone 56 87 71 down
(b) zone 49 car battery 568771 down
(c) battery car 49 zone 56 87 71 down
(d) Cannot be determined

Ans: (d)

81. Input: news 7953 glory for 4629 task Which of the following will be step IV?
(a) 29 task 46 news 53 79 glory for
(b) 29 task 46 news 79 53 glory for
(c) 29 news 79 53 glory for461ask
(d) None of these

Ans: (b)

82. Step mo fan input is: 27 tube 34 gas chamber row 7453 Which of the following steps will be the last but one?
(a) VI
(b) VII
(c) VIII
(d) V    

Ans: (d)

83. Step II of an Input: 19 year 85 74 near gone 26 store How many more steps will be required to complete the rearrangement?
(a) Three
(b) Four
(c) Two
(d) Five        

Ans: (b)

84. In a team, eight boys play Chess. The remaining boys, who represent 7 times the square root of the strength of the tem, play Mouth organ. What is the strength of the team?
(a) 36
(b) 16
(c) 64
(d) 100

Ans. (c)

85. Cost price of 15 Computers is equal to the selling price of 20 Computers, then the loss percent would be
(a) 25%
(b) 20%
(c) 30%
(d) 45%

Ans. (a)

86. The length of the minute hand of a clock is 12 cm. Then the area swept by the minute hand in one minute will be
(a) 22•12 cm2
(b) 23•10 cm2
(c) 24•12 cm2
(d) None of the above

Ans. (d)

87. How many natural numbers between 1 and 900 are multiples of any of the numbers 2, 3, or 5?
1. 240 2. 250 3. 270 4. 300
(a) 1 & 2
(b) only 1
(c) 2, 3 & 4
(d) none of the above

Ans. (c)

88. If Yellow is called Green, Green is called Black, Black is called White, White is called Red, Red is called Blue, Blue is called Orange then what is the colour of milk ?
(a) White
(b) Black
(c) Red
(d) Green

Ans. (c)

89. Four of the following five are alike in a certain way and so form a group. Which is the one that does not belong to the group?
(a) Silver
(b) Gold
(c) Nickel
(d) Brass

Ans. (d)

90. In a certain code EXTRA is written as % 5 # 73 and NOSE is written as 4 @ 8%; how is STORE written in that code?
(a) 5#@7%
(b) 8#@7%
(c) 8#@%5
(d) 8@75%

Ans. (b)

91. In a row of children facing North, Shamika is third to the right of Nikhil who is seventeenth from the right end of the row. Ravi is fifth to the left of Shamika and is twentieth from the left end. Total how many children are there in the row?
(a) 39
(b) 38
(c) 37
(d) Cannot be determined

Ans. (b)

92. If the function f : I –? R, f(x) = log x then the value of f(x) + f(y) is
(a) f(xy)
(b) f(x+ y)
(c) f (x/y)
(d) f(y/x)

Ans. (c)

93. What is the median of 2, 4, 6, 8, 10, 12, 14, 16?
(a) 8
(b) 9
(c) 10
(d) 11

Ans. (b)

94. The average wages of a worker during a fortnight comprising 15 consecutive working days was Rs.90 per day. During the first 7 days, his average wages was Rs.87/day and the average wages during the last 7 days was Rs.92 /day. What was his wage on the 8th day?
(a) 83
(b) 92
(c) 90
(d) 97

Ans. (d)

95. The average of 5 quantities is 6. The average of 3 of them is 8. What is the average of the remaining two numbers?
(a) 6.5
(b) 4
(c) 3
(d) 3.5

Ans. (c)

96. Examine the following relationships among members of a family of six persons A, B, C, D, E and F.
1. The number of males equals that of females
2. A and E are sons of F
3. D is the mother of two, one boy and one girl
4. B is the son of A
5. There is only one married couple in the family at present

Which one of the following inferences can be drawn from the above?
(a) A, B and C are all females
(b) A is husband of D
(c) E and F are children of D
(d) D is the granddaughter of F

Ans. (b) 

97. The age of a man is three times the sum of the ages of his two sons. Five years hence, his age will be double of the sum of the ages of his sons. The father’s present age is
(a) 40 years
(b) 45 years
(c) 50 years
(d) 55 years

Ans. (b) 

98. A car travels the first one-third of a certain distance with a speed of a certain distance with a speed of 10km/hrs. The next one-third of distance with a speed of 20 km/hrs. and the last one-third distance with a speed of the car for the whole journey  is :
(a) 18km/hrs.
(b) 24km/hrs.
(c) 30km/hrs.
(d) 36km/hrs.

Ans. (a) 

99. Find the coordinates of the point which divides the line joining (5, -2) and (9, 6) internally in the ratio 1 : 3.
(a) (6, 0)
(b) (6, 3)
(c) (0, 6)
(d) (3, 6)

Ans. (a)

100. Find the number of triangles in an octagon.
(a) 326
(b) 120
(c) 56
(d) Cannot be determined

Ans. (c)

IAS Prelims CSAT 2011, Model Question Paper (Solved) Set – 2

1. If the angle of triangle are in the ratio of 4:3:2, then the triangle
(a) Is obtuse angled triangle
(b) Has one angle greater than 800
(c Is a right triangle
(d) Is acute angled triangle

Ans. (d)

2. The average monthly income of a person in a certain family of 5 is Rs. 1000/-. What will be the monthly average income of a person in the same family if the income of one person increased by Rs. 12000/- per year?

(a) 1200/-
(b) 1600/-
(c) 2000/-
(d) 3400/-

Ans. (a)

3. In the series POQ, SRT, VUW,—— , the blank space refers to

(a) XYZ
(b) XZY
(c) YXZ
(d) YZX

Ans. (c)

4. The difference between the compound interest and the simple interest on a certain sum at 12% p.a. for two years is Rs.90. What will be the value of the amount at the end of 3 years?

(a) 9000
(b) 6250
(c) 8530.80
(d) 8780.80

Ans. (d)

5. Vijay invested Rs.50,000 partly at 10% and partly at 15%. His total income after a year was Rs.7000. How much did heinvest at the rate of 10%?

(a) Rs.40,000
(b) Rs.40,000
(c) Rs.12,000
(d) Rs.20,000

Ans. (b)

6. A sum of money invested for a certain number of years at 8% p.a. simple interest grows to Rs.180. The same sum of money invested for the same number of years at 4% p.a. simple interest grows to Rs.120 only. For how many years was the sum invested?

(a) 25 years
(b) 40 years
(c) 33 years and 4 months
(d) Cannot be determined

Ans. (a)

Instructions (Q 7-Q 16): Read the following passage carefully and answer the questions given below it.
“Life is not just party and pleasure; it is also pain and despair. Unthinkable things happen. Sometimes everything turns upside down. Bad things happen to good people. Some things are beyond control, such as physical disability and birth defects. We cannot choose our parents or the circumstances of our birth. So if the ball bounced that way, sorry. But what do we do from here; cry or take the ball and run? That is a choice we have to make. On a clear day, there are hundreds of boats sailing in all different directions in a lake. How come? Even though the wind is blowing in one direction, the sailboats are going in different directions. What is the difference? It depends on the way the sail is set, and that is determined by the sailor. The same is true of our lives. We can’t choose the direction of the wind, but we can choose how we set the sail. We can choose our attitude even though we cannot always choose our circumstances. The choice is either to act like a victor or a victim. It is not our position but our disposition that determines our destiny. It takes both rain and sunshine to create a rainbow. Our lives are no different. There is happiness and sorrow. There is the good and the bad; dark and bright spots. If we can handle adversity, it only strengthens us. We cannot control all the events that happen in our lives, but we can control how we deal with them. Richard Blechnyden wanted to promote Indian tea at the St. Louis World fair in 1904. It was very hot and no one wanted to sample his tea. Blechnyden saw that all the other iced drinks were doing flourishing business. It dawned on him to make his tea into an iced drink, mix in sugar and sell it. He did and people loved it. That was the introduction of iced tea to the world. When things go wrong, as they sometimes will, we can react responsibly or resentfully. Human beings are not like an action which has no choice. An action cannot decide whether to become a giant tree or to become food for the squirrels. Human beings have choices. If nature gives us a lemon, we have a choice: either cry or make lemonade.”

7. “We cannot choose our parents or the circumstances of our birth” – by this statement what does the author mean?

(a) Some things do not take place according to our choice
(b) There are certain things in which we do not have any reach
(c) Some things in this world are pre-determined
(d) All of the above

Ans. (d)

8. By what the author tells us that choice is ours?

(a) We can cry
(b) We can take the ball and run away
(c) We can either cry or can take the ball and run away
(d) None of the above

Ans. (c)

9. The opposite of the word ‘victor’?

(a) Loser
(b) Winner
(c) Victim
(d) Warrior

Ans. (a)

10. What determines our destiny?

(a) The way that we normally think and behave, that shows what type of person we are
(d) The position we hold
(c) Our being a winner
(d) All of the above

Ans. (a)

11. The best title for the above passage could be –
(a) Right Choice
(b) Pre-determination
(c) Choices defines our lives
(d) Life is all about making the right choices
Ans. (d)
12. According to the author, what still can be chosen if not everything in life?
(a) Circumstances
(b) Attitude
(c) direction of life
(d) None of these
Ans. (b)
13. On what has the author emphasized?
(a) Destiny
(b) Choice
(c) Action
(d) All of these
Ans. (b)
14. By the example of Blechnyden’s Iced Tea which message does the author give?
(a) Choice is ours
(b) Destiny is pre-determined
(c) We have fix actions; we can not do beyond that
(d) None of the above
Ans. (a)
15. What does actually strengthen us?
(a) Making choices
(b) Handling adversity
(c) Determining destiny
(d) Working beyond capacity
Ans. (b)
16. The synonym of ‘flourishing’ is –
(a) Spoiling
(b) Drowning
(c) Booming
(d) Hiding
Ans. (c)
17. During a 5-day festival, the number of visitors tripled each day. If the festival opened on a Thursday with 345 visitors, what was the attendance on that Sunday?
(a) 345
(b)1,035
(c)1,725
(d) 9,315
Ans. (d)
18. Which of the following has the least value?
(a) 0.27
(b) 1/4
(c) 3/8
(d) 11%
Ans. (d)
19. Which year did the same number of boys and girls attend the conference?
(a) 1995
(b) 1996
(c) 1997
(d) 1998
Ans. (a)
20. Which two years did the least number of boys attend the convention?
(a) 1995 and 1996
(b) 1995 and 1998
(c) 1996 and 1997
(d) 1997 and 1994
Ans. (a)
21. Examine the following three statements :
1. Proceesed meat is a perishable food
2. All perishable foods are packed in sealed tins
3. Sealed tins sometimes to do not contains processed meat
Which one of the following inferences can be drawn from the above statements?
(a) Sealed tins always contain perishable food
(b) Processed meat is sometimes not packed in sealed tins
(c) Proceessed meat is always packed in seald tins
(d) Non-Perishable foods are never packed in seald tins
Ans. (c)
22. Production of Rice and Wheat
(In millions of Tonnes)
Year Rice Wheat Percentage of Wheat to Rice
1950-51 20.58 6.46 31.4
1960-61 34.58 11.00 31.8
1970-71 42.22 23.83 56.4
1980-81 53.63 36.31 67.7
1990-91 74.29 55.14 74.2
1994-95 81.81 65.77 80.4
1995-96 79.62 62.62 78.6
The above table indicates the performance in India in rice and wheat production from 1950-51 to 1995-96. Which of the following conclusions arrived at from the above table would be valid?
1. Record production of rice as well as wheat has been in 1994-95
2. The ratio of wheat to rice production seems to have steadily increased over 16 years
3. Wheat has not been popular among the Indian population before 1980.
4. India became self-sufficient in rice and wheat only after 1990.
Select the correct answer using the codes given below
Codes:
(a) 1 and 2      (b) 1, 2, 3 and 4
(c) 3 and 4      (d) None
Ans. (a)
23. A father left a will of Rs.35 lakhs between his two daughters aged 8.5 and 16 such that they may get equal amounts when each of them reach the age of 21 years. The original amount of Rs.35 lakhs has been instructed to be invested at 10% p.a. simple interest. How much did the elder daughter get at the time of the will?
(a) Rs. 17.5 lakhs
(b) Rs. 21 lakhs
(c) Rs. 15 lakhs
(d) Rs. 20 lakhs
Ans. (b)
24. What will Rs.1500 amount to in three years if it is invested in 20% p.a. compound interest, interest being compounded annually?
(a) 2400
(b) 2592
(c) 2678
(d) 2540
Ans. (b)
25. What is the area of the largest triangle that can be fitted into a rectangle of length ‘l’ units and width ‘w’ units?
(a) lw/3
(b) (2lw)/3
(c) (3lw)/4
(d) (lw)/2
Ans. (d)
26. Which of the following is inCorrect?
(a) An incentre is a point where the angle bisectors meet.
(b) The median of any side of a triangle bisects the side at right angle.
(c) The point at which the three altitudes of a triangle meet is the orthocentre
(d) The point at which the three perpendicular bisectors meet is the centre of the circumcircle.
Ans. (b)
27. How long will it take for a sum of money to grow from Rs.1250 to Rs.10,000, if it is invested at 12.5% p.a simple interest?
(a) 8 years
(b) 64 years
(c) 72 years
(d) 56 years
Ans. (d)
28. Rs. 5887 is divided between Shyam and Ram, such that Shyam’s share at the end of 9 years is equal to Ram’s share at the end of 11 years, compounded annually at the rate of 5%. Find the share of Shyam.
(a) 2088
(b) 2000
(c) 3087
(d) None of these
Ans. (c)
Directions (Q 28-Q 32): Study the following information carefully to answer these questions. Eight members A, B, C, 0, E, F, G and H belonging to three families X, Y, Z go for weekend outing in three different cars I, II, III. Four out of the eight members are females. Members of any one family travel in different cars. Each car has at least one male and one female member. Each family has at least two members. A belongs to family Y and he travels in car III. D is wife of E and they travel in cars I and II respectively. H is son of B, who is wife ofG, and they belong to family Z. C is daughter of F, who is wife of A. C travels in car II. G does not travel with F.
29. Which of the following groups of persons travels in car I?
(a) 0, F,G
(b) D,E,G
(c) D,G, H
(d) D, F, H
Ans. (d)
30. Which of the following members of families Y and Z travel in different cars?
(a) F,G
(b) C,G
(c) F,H
(d) None of these
Ans. (a)
31. Which of the following groups of persons is a group of all females?
(a) B,D,G
(b) A,B,C
(c) B,E,F
(d) None of these
Ans. (d)
32. Which of the following members of families X and Y travel in the same car?
(a) C,F
(b) D,F
(c) C,0
(d) F, E
Ans. (b)
33. When a student weighing 45 kgs left a class, the average weight of the remaining 59 students increased by 200g. What is the average weight of the remaining 59 students?
(a) 57 kgs
(b) 56.8 kgs
(c) 58.2 kgs
(d) 52.2 kgs
Ans. (a)
34. Three math classes: X, Y, and Z, take an algebra test.
The average score in class X is 83.
The average score in class Y is 76.
The average score in class Z is 85.
The average score of all students in classes X and Y together is 79.
The average score of all students in classes Y and Z together is 81.
What is the average for all the three classes?
(a) 81
(b) 81.5
(c) 82
(d) 84.5
Ans. (b)
35. The average of 5 quantities is 10 and the average of 3 of them is 9. What is the average of the remaining 2?
(a) 11
(b) 12
(c) 11.5
(d) 12.5
Ans. (c)
36. A stairway 10ft high is such that each step accounts for half a foot upward and one-foot forward. What distance will an ant travel if it starts from ground level to reach the top of the stairway?
(a) 30 ft
(b) 33 ft
(c) 10 ft
(d) 29 ft
Ans. (d)
37. Each interior angle of a regular polygon is 120 degrees greater than each exterior angle. How many sides are there in the polygon?
(a) 6
(b) 8
(c)12
(d) 3
Ans. (c)
38. A and B are two points with the co-ordinates (-2, 0) and (0, 5). What is the length of the diagonal AC if AB form one of the sides of the square ABCD?
(a) units
(b) units
(c) units
(d) units
Ans. (b)
39. The average weight of a class of 24 students is 36 years. When the weight of the teacher is also included, the average weight increases by 1kg. What is the weight of the teacher?
(a) 60 kgs
(b) 61 kgs
(c) 37 kgs
(d) None of these
Ans. (b)
40. The average of 5 quantities is 10 and the average of 3 of them is 9. What is the average of the remaining 2?
(a) 11
(b) 12
(c) 11.5
(d) 12.5
Ans. (c)
Instruction (Q. 40-Q 43): Four question given below are based on the following information.  Answer the questions after reading the information carefully.
(i) Nine friends A, B,C, D, E, F, G, H, and I are sitting on a bench in the classroom.
(ii) ‘C’  who is at immediate right of ‘D’ is third to the right of ‘E’.
(iii) ‘B’ is at one end.
(iv) ‘H’ is nearest neighbour of ‘F’ and ‘G’
(v) ‘F’ is third to the left of ‘B’
(vi) ‘A’ is at immediate left of ‘F’
41. Which of the following statement is correct on the basis of above order of sitting?
(a) D and H Have three people sitting between them.
(b) ‘B’ is fourth to the right of F.
(c) ‘I’  is at immediate left of A.
(d) E and C are Neighbours.
Ans. (a)
42. Which of the following groups of friends is sitting at the left of ‘C’?
(a) BIG
(b) AFH
(c) EID
(d) BAG
Ans. (c)
43. Which of the following is at the other end i.e. other that that end on which ‘B’ is sitting?
(a) G
(b) H
(c) I
(d) E
Ans. (d)
44. Who is sitting exactly in the middle of the bench?
(a) H
(b) C
(c) A
(d) G
Ans. (c)
45. A 400 meter long train crosses and 800 meter long platform in 1 min 20 sec. What is the speed of the train?
(a) 48 km/hr
(b) 60 km/hr
(c) 72 km/hr
(d) 54 km/hr
Ans. (d)
46. If the root of equation px2 + qx +r = 0 is double of the other root, which one the following is correct?
(a) 2 q2 = 9 pr
(b) 2 q2 = 9
(c) 4 q2 = 9 r
(d) 9 q2 = 2 pr
Ans. (a)
47. A takes 4 days and B takes 5 days to finish a job. If both of them work together on the same job, what proportion of the work is done by A?
(a) 4/9
(b) 5/9
(c) 6/9
(d) 7/9
Ans. (b)
48. The 3rd term of a Geometric progression is 36 and its 6th term is 288. Its 8th term will be:
(a) 784
(b) 576
(c) 1152
(d) 2302
Ans. (c)
49. In a two digit number, the sum of the digits is 8. If 54 is subtracted from this number, its digits interchange themselves. What is the product of the digits of the number?
(a)  7
(b) 12
(c) 16
(d) 0
Ans. (a)
50. The difference between simple interest and compound interest on certain amount for 2 years at the same rate of interest is Rs. 18. If the rate doubled, what will be the difference?
(a) Rs. 9
(b) Rs. 36
(c) Rs. 72
(d) Rs. 27
Ans. (c)
51. A dog is tied to a pole by a long chain. Keeping the chain fully stretched, the dog moves along a circular path covering 132 m, subtends an angle of 90 degree at the centre, Neglecting the portion of the chain used in typing, find the length of the chain.
(a) 70 m
(b) 78 m
(c) 84 m
(d) 88 m
Ans. (c)
52. A merchant buys same quantities of two types of toys – one at the rate of 3 toys for Rs. 10 and the other, at the rate of 5 toys for Rs. 20. At what price he must sell all the toys so that he has no profit and no loss in the  transaction?
(a) 8 toys for Rs. 30
(b) 3 toys for Rs. 11
(c) 5 toys for Rs. 18
(d) 8 toys for Rs. 22
Ans. (b)
53. Find the equation of a line whose intercepts are twice of the line 3x – 2y – 12 = 0
(a) 3x – 2y = 24
(b) 2x – 3y = 12
(c) 2x – 3y = 24
(d) None of these
Ans. (a)
54. The difference between the compound interest and the simple interest on a certain sum at 12% p.a. for two years is Rs.90. What will be the value of the amount at the end of 3 years?
(a) 9000
(b) 6250
(c) 8530.80
(d) 8780.80
Ans. (d)
55. Vijay invested Rs.50,000 partly at 10% and partly at 15%. His total income after a year was Rs.7000. How much did heinvest at the rate of 10%?
(a) Rs.40,000
(b) Rs.40,000
(c) Rs.12,000
(d) Rs.20,000
Ans. (b)
56. The average weight of a class of 24 students is 36 years. When the weight of the teacher is also included, the average weight increases by 1kg. What is the weight of the teacher?
(a) 60 kgs
(b) 61 kgs
(c) 37 kgs
(d) None of these
Ans. (b) 

Instructions for the following Question:  Read the following information to answer the question given below.

The following conditions have been decided for the selection of Research Assistants. The candidate must:
(i) Be less than 35 years of age.
(ii) Be graduate in Psychology, Education or Commerce or Postgraduate in any other subject.
(iii) Secure 50% marks in entrance examination.
(iv) Secure A or O grade in Interview.
(v) Have  passed a certificate course in Methodology Research.
(vi) Have recommendation of any one lecturer. If a candidate fulfills all the conditions, except
(a) (i), but if he is registered for Ph.D., his case is to be kept in waiting list.
(b) (iii), but if he has secured 45% marks in entrance examination and O grade in interview, his case is to be referred to the Dean.
(c) (v), but if he has qualified certificate course in Statistics, his case is to be referred to the Head Of Department.
You have to decide in each of the following cases that what is the possibility of selection of the candidate? If the information is not sufficient to take any decision then your answer will be ‘data inadequate’. This information has been given to you as on 15.12.1996.
57. Raman has secured 60% marks in graduation with commerce subject. He has got specialization in certificate course in Methodology Research and he also has the recommendation from the lecturer under whom he is registered for Ph.D. His date of birth is 9.12.61. He has secured 68% marks in entrance examination and got A grade in interview.
(a) Will not be selected.
(b) Data inadequate
(c) Will be selected.
(d) Will be kept in waiting List.
Ans. (d)
58. If ‘’ means ‘–’; ‘–’ means ‘x’; ‘x’ means ‘+’ and ‘+’
means ‘’, then 40 × 120 80 – 40 + 20 = ?
(a) 80
(b) 120
(c) 60
(d) 0
Ans: (d)
59. A businessman makes a profit of 20% on the sale of leather. If he were to add 10% artificial matter to the leather, by what percent would his profit increase?
(a) 25%
(b) 60%
(c) 40%
(d) 45%
Ans. (b)
60. In a team, eight boys play Chess. The remaining boys, who represent 7 times the square root of the strength of the tem, play Mouth organ. What is the strength of the team?
(a) 36
(b) 16
(c) 64
(d) 100
Ans. (c)
61. Cost price of 15 Computers is equal to the selling price of 20 Computers, then the loss percent would be
(a) 25%
(b) 20%
(c) 30%
(d) 45%
Ans. (a)
62. How many natural numbers between 1 and 900 are not multiples of any of the numbers 2, 3, or 5?
(a) 240
(b) 250
(c) 270
(d) 300
Ans. (a)
63. The age of a man is three times the sum of the ages of his two sons. Five years hence, his age will be double of the sum of the ages of his sons. The father’s present age is
(a) 40 years
(b) 45 years
(c) 50 years
(d) 55 years
Ans. (b)
Instruction (Q 62-Q 63): In the two questions given below, a statement followed by two arguments I and II has been given. You have to decide which argument is strong and which one is weak?
Give your answer as:
(a) If only argument I is strong;
(b) If only  agreement II is strong;
(c) If neither argument I nor argument II is strong;
(d) If both argument I and argument II are strong.
64. Statement:
Should freelance work programme be stopped at private companies?
Argument:
I. Yes, the mutual interaction of the employee and office environment helps in overall quality of work.
II. No, such demand comes from that group only who don’t have any idea of work loads and pressure.
Ans. (d)
65. Statement:
Should usage of pesticides in agriculture be banned in India?Agriculture be banned in India?
Arguments:
I. Yes, because pesticides pollute the environment and are also harmful for the ecology.
II. No, the safety of crops is not possible without these and food products will become very expensive due to lack of food production.
Ans. (d)
66. If the numbers representing volume and surface area of a cube are equal, then the length of the edge of the cube in terms of the unit of measurement will be
(a) 3
(b) 4
(c) 5
(d) 6
Ans. (d)
67. In a group of travelling in a bus, 6 persons can speak Tamil, 15 can speak Hindi and 6 can speak Gujrati. In that group none can speak any other language. If 2 persons in the group can speak two languages and one person can speak all the three languages, then how many persons are there in the group?
(a) 21
(b) 22
(c) 23
(d) 24
Ans. (d)
68. A boat which has a speed of 5 km/hr in still water crosses a river of width 1 km along the shortest possible path in 15 minutes. The velocity of the river water in km/hr is
(a) 1
(b) 3
(c) 4
(d) √41
Ans. (a)
69. When three coins are tossed together the probability that all coins have the same face up, is
(a) 1/3
(b) 1/6
(c) 1/8
(d) 1/12
Ans. (c)
Directions (Q 68-Q 72):,Study the following arrangement carefully and answer the questions given below:
H93P$KE%4FR I U@WG2MI5BQZ6©*N&8VJ
70. If all the symbols and numbers are dropped from the above arrangement, which of the following will be the fourteenth from the right end?
(a) M
(b) K
(c) W
(d) E
Ans. (d)
71. What should come in place of the question mark (?) in the following series based on the above arrangement?
_PK% RUW M5Q ?
(a) ©N8
(b) ©N15    
(c) 15*15
(d) 6*8
Ans. (a)
72. How many such numbers are there in the above arrangement each of which is immediately preceded by a letter and immediately followed by a symbol?
(a) None
(b) One
(c) Two
(d) Three
Ans. (b)
73. Which of the following is the seventh-to the left of the twentieth from the left end of the above arrangement?
(a) U
(b)S    
(c)M
(d)N
Ans. (a)
74. How many such consonants are there in the above arrangement each of which is immediately followed by a number but not immediately preceded by a number?
(a) None
(b) One 
(c) Two
(d) More than three
Ans. (d)
75. Left pan of faulty weighs 100gram more than is right pan. A shopkeeper keeps the weight measure in the left pan while buying goods but keeps it in the right pan while selling his goods. He uses only 1kg weight measure. If he sells his goods at the listed cost price, What is his gain?
(a) 200/11%
(b) 100/11%
(c) 1000/9%
(d) 200/9%
Ans. (a)
76. Societies have been increasing in complexity from folk to industrial. Under this circumstances, according to you social control is more likely to be invested in –
(a) family
(b) school
(c) state
(d) religious structures
Ans. (c)
Instructions (Q 77-Q 86): Read the following passage carefully and answer the questions given below it.
“The big difference between the ideas of Aristotle and those of Galileo and Newton is that Aristotle believed in a preferred state of rest, which any body would take up if it were not driven by some force or impulse. In particular, he thought that the earth was at rest. But it follows from Newton’s laws that there is no unique standard of rest. One could equally well say that body A was at rest and body B was moving at constant speed with respect to body A, or that body B was at rest and body A was moving. For example, if one sets aside for a moment the rotation of the earth and its orbit round the sun, one could say that the earth was at rest and that a train on it was travelling north at ninety miles per hour or that the train was at rest and the earth was moving south at ninety miles per hour. If one carried out experiments with moving bodies on the train, all Newton’s laws would still hold. For instance, playing Ping-pong on the train, one would find that the ball obeyed Newton’s laws just like a ball on a table by the track. So there is no way to tell whether it is the train or the earth that is moving.
The lack of an absolute standard of rest meant that one could not determine whether two events that took place at different times occurred in the same position in space. For example, suppose our Ping-Pong ball on the train bounces straight up and down. Hitting the table twice on the same spot one second apart. To someone on the track, the two bounces would seem to take place about forty meters apart, because the train wood have traveled that far down the track between the bounces. The nonexistence of absolute rest therefore meant that one could not give an event an absolute position in space, as Aristotle had believed. The position of events and the distance between them would be different far a person of the train and one on he track, and there would be no reason to prefer one person’s position to the other’s.
Newton was very worried by this lack of absolute position, or absolute space, as it was called, because it did not accord with his idea of an absolute God. In fact, he refused to accept lack of absolute space, even though it was implied by his laws. He was severely criticized for this irrational belief by many people, most notably by Bishop Berkeley, a philosopher who believed that all material objects and space and time are an illusion. When the famous Dr. Johnson was told of Brekeley’s opinion, he cried, “I refute it thus!” and stubbed his toe on a large stone.
Both Aristotle and Newton believed in absolute time. That is, they believed that one could unambiguously measure the interval of time between two events, and that this time would be the same whoever measured it, provided they used a good clock. Time was completely separate from and independent of space. This is what most people would take to be the commonsense view. However, we have had to change our ideas about space and time. Although our apparently commonsense nations work well when dealing with things like apples, or planets that travels comparatively slowly, they don’t work at all things moving at or near the speed of light.”
77. Consider the following statements:
1. Newton is a firm beliver of both absolute time and a preferred state of rest
2. Aristotle believer in absolute time
3. Newton believes in absolute time
4. Galileo believes in a preferred state of rest
(a) 1 Only
(b) 1 & 2
(c) 2 & 3
(d) 1, 2, 3 & 4
Ans. (c)
78. According to the author, what is the need of space?
1. To set an absolute standard of rest
2. To determine rest and motion
3. To determiner whether two events taking place at the same time takes place in the same space also
4. To determine the defference between time and space
(a) 1 & 2
(b) 1, 2 & 3
(c) 3 & 4
(d) 1, 2, 3 & 4
Ans. (c)
79. According to Aristotle-
(a) Position of events and distances between them an different
(b) Space is nonexistent
(c) All nonexistent bodies have absolute rest
(d) None of the above
Ans. (a)
80. From the passage what can we infer?
(a) There is no rest
(b) There is rest
(c) There is absolute rest
(d) There is no absolute rest
Ans. (d)
81. What seems contradictory in Newton’s laws?
(a) Newton’s concept of Absolute God
(b) Concept of Absolute space
(c) Concept of rest
(d) None of these
Ans. (b)
82. Who discards material objects and space and time?
(a) Newton
(b) Aristotle
(c) Barkeley
(d) Galileo
Ans. (c)
83. Consider the following statements according to the information provided in paragraph three?
(a) On Barkeleys opinion, Dr. Johnson had no remarks
(b) Dr. Johnson criticized Barkeley vehemently
(c) Dr. Johnson accepted Barkeley’s view
(d) Dr. Johnson was indifferent
Ans. (b)
84. “The interval of time between two events can be measured unambiguously.”-this implies-
(a) Absolute space
(b) Absolute time
(c) Both Absolute time and absolute space
(d) Neither absolute time non absolute space
Ans. (b)
85. The term “unambiguous” means?
(a) Clean and proper
(b) Perfect
(c) Confusing or not difined
(d) Well difined
Ans. (c)
86. What does the author openion in the passage?
(a) Time is completely separated and independent from space
(b) Time and space and interdependent
(c) Time is dependent on space and not vice-versa
(d) Space is dependment on time and vice-versa
Ans. (a)
87. A sum of money invested for a certain number of years at 8% p.a. simple interest grows to Rs.180. The same sum of money invested for the same number of years at 4% p.a. simple interest grows to Rs.120 only. For how many years was the sum invested?
(a) 25 years
(b) 40 years
(c) 33 years and 4 months
(d) Cannot be determined
Ans. (a)
88. How long will it take for a sum of money to grow from Rs.1250 to Rs.10,000, if it is invested at 12.5% p.a simple interest?
(a) 8 years
(b) 64 years
(c) 72 years
(d) 56 years
Ans. (d)
89. Rs. 5887 is divided between Shyam and Ram, such that Shyam’s share at the end of 9 years is equal to Ram’s share at the end of 11 years, compounded annually at the rate of 5%. Find the share of Shyam.
(a) 2088
(b) 2000
(c) 3087
(d) None of these
Ans. (c)
90. An equilateral triangular plate is to be cut in to n number of identical small equilateral triangular plates. Which of the following can be possible value of n?
(a) 196
(b) 216
(c) 256
(d) 296
Ans. (c)
91. Find the area of the sector covered by the hour hand after it has moved through 3 hours and the length of the hour hand is 7cm.
(a) 1. 77 sq.cm
(b) 2. 38.5 sq.cm
(c) 3. 35 sq.cm
(d) 4. 70 sq.cm
Ans. (b)
92. Left pan of faulty weighs 100gram more than is right pan. A shopkeeper keeps the weight measure in the left pan while buying goods but keeps it in the right pan while selling his goods.He uses only 1kg weight measure. If he sells his goods at the listed cost price, What is his gain?
(a) 200/11%
(b) 100/11%
(c) 1000/9%
(d) 200/9%
Ans. (a)
93. Societies have been increasing in complexity from folk to industrial. Under this circumstances, according to you social control is more likely to be invested in –
(a) family
(b) school
(c) state
(d) religious structures
Ans. (c)
94. Each interior angle of a regular polygon is 120 degrees greater than each exterior angle. How many sides are there in the polygon?
(a) 6
(b) 8
(c) 12
(d) 3
Ans. (c)
95. In the following question, the student’s clinical judgement is assessed:
A 28 year old woman with one child has taken anti-thyroid drugs for 6 months for thyrotoxicosis. She has a friend who has been successfully treated with radioiodine. She finds she frequently forgets to take her drugs and wants to stop them to have radio-iodine treatment.
1. She should be told that because of her age radio-iodine is best avoided.
2. The problems associated with radio-iodine should be discussed with her.
3. Surgery as a possible alternative should be discussed with her.
4. She should be advised that some form of further treatment is required.
5. You should find out more about her friend’s treatment.
(a) 1 only
(b) 2, 3 & 4
(c) 1 & 5
(d) 1, 2 & 3
Ans. (b)
96. In a certain store, the profit is 320% of the cost. If the cost increases by 25% but the selling price remains constant, approximately what percentage of the selling price is the profit?
(a) 30%
(b) 70%
(c) 100%
(d) 250%
Ans. (b)
Directions for the following questions from Question No 97 to Question No 100:
The following passage in this section is followed by questions based on the content of the reading passage. Read the passage carefully and chose the best answer to each question below.
“But man is not destined to vanish. He can be killed, but he cannot be destroyed, because his soul is deathless and his spirit is irrepressible. Therefore, though the situation seems dark in the context of the confrontation between the superpowers, the silver lining is provided by amazing phenomenon that the very nations which have spent incalculable resources and energy for the production of deadly weapons are desperately trying to find out how they might never be used. They threaten each other, intimidate each other and go to the brink, but before the total hour arrives they withdraw from the brink.”
97. What is the synonym of the word, confrontation?
(a) Being face to face
(b) Involving in a clash
(c) Fair relationship
(d) None of the above
Ans: (a)
98. What is the crux of the passage?
(a) Man’s soul and spirit can not be destroyed by superpowers.
(b) Man’s soul and spirit are immortal.
(c)Man’s safety is assured by the delicate balance of power in terms of nuclear weapons.
(d) Human society will survive despite the serious threat of total annihilation.
Ans : (d)
99. The phrase ‘Go to the brink’ in the passage means –
(a) Retreating from extreme danger.
(b) Declare war on each other.
(c) Advancing to the stage of war but not engaging in it.
(d) Commit suicide.
Ans : (c)
100. What is the author’s opinion?
(a) Nations in possession of huge stockpiles of lethal weapons are trying hard to avoid actual conflict.
(b) Huge stockpiles of destructive weapons have so far saved mankind from a catastrophe.
(c) Mankind is heading towards complete destruction.
(d) There is a Silverlining over the production of deadly weapons.
Ans : (a)

IAS Prelims History (Prelims)/ quiz

 1. The Mughal emperor taken prisoner by
Nadir Shah was
a. Bahadur Shah
b. Jahandar Shah
c. Farrukh Siyar
d. Muhammad Shah
2. Who among the following were among
those who caused major uprisings against
Murshid Quil Khan?
1. Sitaram Ray
2. Udai narayan
3. Chulam Muhammad
4. Saadat Khan
a. 1 and 2
b. 2 and 4
c. 1, 2 and 3
d. 1 and 4
3. Haidar Ali established a modern arsenal at
a. Mysore
b. Srirangappattanam
c. Dindigal
d. Arcot
4. Fakir Azizudding and Dewan Dina Nath
were ministers under
a. Muhammad Shah
b. Suraj mal
c. Muhammad Khan Bangash
d. Ranjit Singh
5. The Bental Nawab who collected revenue
directly from the ryots through his agents
called ijaradars was
a. Aliwardi Khan
b. Murshid quli Khan
c. Shujauddin
d. Sarfaraz Khan
6. Who among the following were the allies
of Mr Qusim in the battle of Buxar in
1964?
a. Muzaffar Jung and Muhammad Khan
Bangash
b. Shuja-ud Dulah and Muhammad Ali
c. Shuja-ud Daulah and Shah Alam II
d. Shah Alam II and Muhammad Ali.
7. Which one of the following is correctly
matched?
a. Orissa famine of 1866: Sir John
Lawrence
b. Agricultural Research Institute: Lord
Hardinge at Pusa
c. Development of Irrigation: Lord
Auckland
d. Bengal Famine of 1943: Lord Irwin
8. Which one of the following pairs is not
correctly matched
a. Rytwari Settlement: Madras
b. Permanent Settlement: Bengal
c. Mahalwari Settlement: North Western
province
d. Talukdari System: Bombay
9. What is the correct sequence of the
following?
1. arms Act
2. Ilbert Bill Controversy
3. First Census in India
4. Partition of Bengal
a. 3, 1, 2, 4
b. 3, 2, 1, 4
c. 2, 1, 3, 4
d. 1, 3, 4, 2
10. The chief organizer of the Revolt of 1857
in Bihar was
a. Khan Bahadur Khan
b. Maulavi Ahmadullah
c. Kunwal Singh
d. Rao Sahib
11. What is the correct Chronological
sequence of the following events in the
Revolt of 1857
1. Declaration of Delhi as the
independent capital of the Mughal
empire once again.
2. Revolt in Kanpur and the declaration
of Nana Saheb as Raja under the
Mughal emperor.
3. Hanging of Tantia tope to death
4. The marching of sepoys toDelhi and
their entry is the Red Fort.
a. 4, 1, 2. 3
b. 4. 1. 3. 2
c. 3, 4, 1, 2
d. 2, 3, 4, 1
12. Zeliongrong Movement was organized in
a. Nagaland
b. Manipur
c. Tripura
d. Mizoram
13. Match List-I with List-II and select the
correct answer
List-I
A. Buddhu Bhagat
B. Sido and Knhu’
C. Titu Mir
D. Ganganarayan
E. Shariatullah
List-II
1. Santal rebellion (1855)
2. Bhumij rebellion (1833)
3. Farazi Movement (1831-37)
4. Kol insurrection (1831-32)
5. Barasat Vidroha (1831)
Codes: A B C D E
a. 4 5 2 1 3
b. 2 1 3 2 4
c. 4 1 5 2 3
d. 3 2 5 4 1
14. With a view to encouraging the spread of
western education among the Muslims, the
Muhammadan Literary Society was found
in 1863 at
a. Calcutta
b. Aligarh
c. Delhi
d. Karachi
15. Pagal Panthis sect was founded by
a. Syed Ahmad of Rae Bareli
b. Haji Shariatullah
c. Karam Shah
d. Shah Waliullah
16. What is the correct chronological sequence
of the following event significant in the
history of modern education inIndia?
1. The establishment of the women
university at Poona
2. Carlyle circular
3. Hunter commission
4. Establishment of the Aligarh Muslim
University
a. 1, 2, 3, 4
b. 4, 3, 2, 1
c. 3, 2, 1, 4
d. 1, 3, 4, 2
17. The Central Hindu School at Banaras
which served as a nucleus of the Banaras
Hindu University was established by
a. Madan Mohan Malviya
b. Mrs. Annie Besant
c. Bhagawan Das
d. Swami Shradhananda
18. The Theosophists advocated the revival
and strengthening of
a. Hindusm Zoroastrianism and
Buddhism
b. Hinduism, Buddhism and Islam
c. Hinduism, Buddhism and Jainism
d. Hinduism and Buddhism only
19. Who among the following were associated
with the starting of the Reumai
Mazdayasan?
1. Naoroji Furdenji
2. Dadabhai Naoroji
3. S.S. Bangalee
a. 1 and 2
b. 1 and 3
c. 2 and
d. 1, 2 and 3
20. Rand was killed by the militant nationalist
a. Damodar Chapekar
b. Barindra Ghosh
c. Jatindranath Banerjee
d. Prafulla chaki
21. The Swaraj Party decided to take part in
the Reforms Scheme under the
Government of India Act, 1919 because.
a. It wanted to gain political experience
by forming government.
b. It wanted to expose the hollowness of
the reforms provided in the Act.
c. It wanted to cooperate with the
government.
d. It wanted to curtail the influence of ‘no
changers’
3 of 11
22. The Congress Ministries in the seven
provinces British India resigned in October
1939 because
a. Linlithgow declared emergency in
India at the outbreak of the Second
World War on 3rd September 1939.
b. Gandhi did not want involvement of
India with the violence of war.
c. India as a British colony was
automatically committed to
belligerency by Linlithgow without
bothering to consult the Provincial
congress Ministries or any Indian
leader.
d. The congress working Committee
which met immediately after the
outbreak of the war failed to elicit a
satisfactory reply from Linlithgow
about the war aims of Britain and how
these were going to affect Indiaduring
and after the war
23. Mahatma Gandhi undertook fast unto
death in September, 1932 because.
a. He wanted to expedite the declaration
of complete independence forIndia.
b. There was dissidence in the Congress
c. There was a communal riot
d. He disapproved of the provision of
separate electorates for the depressed
classes in the British Prime Minister
communal Award.
24. As part of the Civil disobedience
Movement there was defiance of forest
laws in.
a. The Madras Presidency and Orissa
b. Maharashtra, Karnataka and the
Central provinces
c. Kerala, Gujarat and Assam
d. Bengal and Orissa
25. Which of the following newspapers
advocated revolutionary terrorism during
the freedom struggle?
1. The Sandhya
2. The Yugantar
3. The Kal
Select the correct answer from the codes
given.
a. 1 and 2
b. 1 and 3
c. 2 and 3
d. 1, 2 and3
26. Match List-I with List-II and select the
correct answer.
List-I (Congress sessions)
A. Lucknow session, 1916
B. Lahore session, 1929
C. Karachi session, 931
D. Faizpur session, 1936
List-II (Ideological formulations)
1. Full democracy
2. congress-Legue pact
3. Purna Swaraj resolution
4. Adoption of fundamental Rights
Codes: A B C D
a. 3 2 4 1
b. 2 3 4 1
c. 2 4 1 3
d. 4 3 2 1
27. The Hindustan Republican Association
founded for organizing and armed
revolution was renamed as theHindustan
Socialist Republican Association under the
leadership of
a. Acharya Narendra Dev
b. Ram Prasad Bismil
c. Chandra Shekhar Azad
d. S. A. Dange
28. The coalition government formed in
Bengal in 1937 consisted of
a. The Muslim League and the Congress
b. The Unionist Party and the Muslim
League
c. The Krashak Praja Party and the
Muslim league
d. The Krashak Praja Party and the
congress.
29. The Cabinet Delegation consisted of
a. Lord Pethic Lawrence, Sir Stafford
Cripps and Mr. Alexander
b. Lord Wavel, Sir Stafford Cripps and
Mr. Alexander
c. Lord Wavell, Lord Pethic Lawrence
and Mr. Alexander.
d. Sir Stafford Cripps, Lord Wavell and
Lord Pethic Lawrence
30. The first session of the All India Trade
Union congress held in Bombaywas
presided over by.
4 of 11
a. Lala Lajpat Rai
b. S.A. Dange
c. N.M.Joshi
d. B.Shiva Rao
31. Which one of the following statements is
correct?
a. The Indian Independence Act. 1947
gave unlimited powers to the
Constituent Assembly of each
Dominion frame and adopt any
Constitution and to repeal any Act of
the British Parliament
b. The Indian Independence Act, 19478
came into force the 15th August, 1947.
c. The Indian Independence Acts, 1947
provided that the district of Sylhet in
Assam would form part of India
d. The Indian independence Act, 1947
made it mandatory that the Constituent
Assembly’s of India and Pakistan
should consist of members directly
elected by the people.
32. Which one of the following is a site of the
Harappan civilization wherein aPersian
Gulf seal has been unearthed
a. Mohenjodaro
b. Dholavira
c. Lothal
d. Kalibangan
33. Agriculture flourished in
a. The Kutch region
b. Afghansitan
c. Baluchistan
d. Sind
34. The crop which does not seem to have
been known to people of the Harappan
Culture is
a. Rice
b. Cotton
c. Ragi
d. barley
35. Which one of the following is correctly
matched?
a. Surkotada : : Ploughed field
b. Harappa : Horse remains
c. Rangpur : Rice husk
d. Chanhudaro : Citadel
36. The largest number of seals of the Harppan
culture are made of
a. Terracotta
b. Faience
c. Agate
d. Steatite
37. Which of the following were exported
from Meluhha, sometimes identified as a
Harappan area?
1. Black wood
2. 2 Ivory
3. gold
4. lead
a. 1, 2 and 3
b. 1, 2 and 4
c. 1, 3 and 4
d. 2, 3 and 4
38. Who among the following was a
philosopher king of Kekaya Janapada
mentioned in the Upanishads?
a. Jabali Pravahana
b. Jamadagni
c. Srutasena
d. Asvapati
39. Early Indian religious texts refer to
conflict between Krishna and
a. Brahma
b. Rudra
c. Indra
d. Veruna
40. The famous conversation between
Uddalaka Aruni and his son Svetaketu
regarding the identity of the Brahaman and
the Atman figures in the
a. Svetasvatara Upanishad
b. Chandogya Upanishad
c. Mundaka Upanishad
d. Mandukya Upanishad
41. Nilalohita, a type of earthenware
mentioned in the Vedic texts, may be
identified with
a. Painted greyware
b. Redware
c. Black and Redware
d. Northern black Polished ware
42. The term used to refer to barren land in the
Vedic texts in
a. Vraja
b. Kulya
c. Suyavas
d. Khilva
43. Yadu and Turvasa referred to the Rigveda
were
a. Two generals
b. Two brothers
c. Two kings
d. Two tribes
44. What is the correct sequence of the
following in the history of South India?
1. Expansion of the Mauryan rule
2. beginnings of the megalithic culture
3. Sangam age
4. Pallava ascendancy
a. 2, 3, 1 , 4
b. 2, 1, 3, 4
c. 2, 1, 4, 3
d. 1, 2, 3, 4
45. Which one of the following is a toll tax
mentioned in early Tamil literature?
a. Paduporul
b. Iduporul
c. Ulgu
d. Uruporal 
46. Upsatha was practiced
a. By windows of the
b. By Buddhist monks in an assembly
c. By the Kapalikas as symbolic
renunciation of pleasure in external
objects
d. By the Tantriks as part of their worship
of Shakti
47. Which of the following believed in the
worship of the Yakshas and Yakshinis?
1. Brahmanism
2. The Kalamukha sect
3. Buddhism
4. Jainism
48. The doctrine that the human personality
consists of five skandhas is associated with
a. Buddha
b. Mahavira
c. Parsvanatha
d. Maskariputra Gosla
49. The concept of the Eight-fold path forms
the theme of
a. Dharma Chakra Pravartana Sutta
b. Divyavadana
c. Dipavamsa
d. Mahaparinibban Sutta
50. Nagarjuna’s Sunjavada is expounded in
a. Yogachara
b. Vaibhashika
c. Madhyamika
d. Sautrantika
51. Purvasailas were a branch of
a. The Sthaviravada sect of Buddhism
b. The Mahasanghika sect of Buddhism
c. The Svetambara sct of Jainism
d. Purva-mimamsakas
52. Which among the following accepts only
Pratyaksha Pramana?
a. Mimamsa
b. Nyaya
c. Samkhya
d. Lokayata
53. biographies of jaina Tirthankaras are found
in
a. Bhagavati sutra
b. Kalpa sutra
c. Niryavali sutra
d. Uvasagadasao
54. Which of the following hills is/are sacred
to the jains?
1. Arbudagiri
2. Satrunjayagiri
3. Chandragiri
4. Urjayantagiri
a. 1 only
b. 1 and 3
c. 2 and 4
d. 1, 2, 3 and 4
55. Among the Panchaviras who belonged to
the Vrishni clan, Samba was the son of
a. Rohini
b. Rukmini
c. Jambavati
d. Devaki
56. Which of the following were ruling
powers in whose records the worship of
Skanda is found mentioned?
1. Stavahanas
2. Yaudheyas
3. Ikshvakus
4. Chedis
6 of 11
a. 1 and 2
b. 1 and 3
c. 2 and 3
d. 3 and 4
57. The term Satvata Vidhi denotes
a. Tantrik practices
b. Bhagavata ritual
c. Mahayana philosophy
d. Mode of Lakulisa worship
58. Who among the following were the
Kushna rulers whose coins bear either
Siva, Siva and bull or one of the emblems
of siva?
1. Huvishka
2. Kanishka 1
3. Kujula Kadphises
4. Wima Kadphises
a. 1, 2 and 3
b. 2, 3 and 4
c. 1, 3 and 4
d. 1, 2 and 4
59. Periyapuranam is a work on
a. Bengal Vaishnavism
b. Tamil Saivism
c. Kashmir Salvism
d. Post-Gupta Saktism
60. Gopis (cowherd girls) became and
important par of the Krishnalegend in art
and literature in
a. the Sunga period
b. the Kushana period
c. the Gupta period
d. the early medieval period
61. Match List – I with II and select the
correct answer
List-I: sources mentioned
A. Greak source
B. Jain source chief
C. Buddhist source
D. Brahamanical source
List–II information about
Chandragupta Maurya, correct or
incorrect, given in source in List I
1. He was born in a humble family
2. he was the son of a Kshatriya
3. he was a Sudra
4. he was the son of a village headman’s
daughter
5. he was a Jain
Codes: A B C D
a. 4 5 3 1
b. 1 3 4 2
c. 1 4 2 3
d. 3 5 2 4
62. Ashoka has been mentioned by name in
his inscriptions at
a. Maski and Gujarra only
b. Dhauli and Raupnath only
c. Nittur, Udegolam, Maski and Gujarra
d. Kandhar, Bairat, Yerragudi and Maski.
63. Which one of the following mentions
Chandragupta Sabha, the council of
Chandragupta Maurya?
a. Arthasastra
b. Mudrarakshasa
c. Mahabhashya
d. Parisistaparvan
64. Which of the following were meant by
Ashoka by the term?
1. Firm attachment to only one’s own
religion
2. Taking care of mother and father
3. No-violence to living beings
4. Moderation in both spending and
saving
a. 1 only
b. 1 and 3
c. 2, 3 and 4
d. 1, 2, 3 and 4
65. The Mauryan ruler who was also known
by the name Amitraghta was
a. Bindusara
b. Dasaratha
c. Sailisuka
d. Brihadratha
66. The official credited with the construction
of the Sudarshan lake in Saurashtra during
the reign of Chandragupta Maurya was
a. Yavfanaraga tusaspha
b. Parnadatta
c. Pahlava Suvisakha
d. Vaisya Pusyagupta
7 of 11
67. Which of speak of the state granaries
instituted to combat the ravages of famine
in pre-Christian times?
a. Rampurva and Lauriya nandangarh
pillar inscriptions
b. Girmar and Jugada rock inscriptions
c. Maski and Bairat minor rock
inscriptions
d. Sohgaura and Mahasthangarh plaque
inscriptions.
68. The Cholas, the Pandays, the Keralaputras
and the Satyaputras are mentioned as his
neighbouring powers by Asoka in
a. Rock Edict II
b. Rock Edict XIII
c. Pillar Edict VII
d. Minor rock Edicts
69. Who among the following are the
Hinsippas mentioned in the Jatakas?
1. wood workers
2. Hunters and trappers
3. Bucher
4. Ivory workers
a. 1 and 2
b. 1 and 3
c. 2 and 3
d. 3 and 4
70. Among the four dynasties listed below
which ones minted coins made of lead?
1. Mauryas
2. Satavahanas
3. Western Kshatrapas
4. Guptas
a. 1 and 2
b. 1, 2 and 4
c. 2, 3 and 4
d. 3 and 4
71. Paloura, mentioned as a port in the early
sources was located on the coast of
a. Kerala
b. Vanga
c. Kalinga
d. Andhra
72. According to the Periplus, sea voyages to
India were undertaken in the month of
Epiphi or
a. October
b. July
c. June
d. December
73. Which of the following wee used for
drawing water irrigation during the post-
Gupta period?
1. Araghtta
2. Vapi
3. Tadaga
4. Pranali
a. 1,2 and 3
b. 1 and 3
c. 2 and 4
d. 1, 2, and 4
74. Coconut cultivation in India was know
from the period
a. Mauryas
b. Satavahanas and Kshtrapas
c. Guptas
d. Pallavas and Western Gangas
75. Which one of the following is not matched
correctly?
a. Angula : Unit of linear measure
b. Veli : Unit for measuring space
c. Kamal : Unit for liquid measure
d. Kalanju : Unit of weight 
76. Which of the following terms mentioned
in early source denote land-measure?
1. Nivartana
2. Kulyavapa
3. Dronavapa
4. Rathya
a. 1 and 2
b. 2 and 3
c. 1, 2 and 3
d. 3 and 4
77. In which one of the following forms of
marriage bride-price was paid by the
groom?
a. Brahma
b. Rakshasa
c. Paisacha
d. Asura
78. Which of the following were considered
untouchable in early India?
1. Charmakara
2. Swarnakara
3. Paraiyar
8 of 11
4. Rathakara
a. 1 and 2
b. 1 and 3
c. 1 and 4
d. 3 and 4
79. Which one of the following is the oldest
Smriti
a. Vishnu Dharmasastra
b. Manu smriti
c. Yajnavalkya smriti
d. Narada smriti
80. Given below are tow statements, one
labeled as Assertion (A) and the other
labeled as Reason (R):
Assertion (A): Gautamiputra Satakami was
proposed to the system of four varnas.
Reason (R): Gautamiputra Satakarni
extended patronage to Buddhist monks.
In the context of the above two statements,
which one of the following is correct?
a. Both A and R are true and R is the
correct explanation of A.
b. Both A and R are true but R is not a
correct explanation of A
c. A is true but R is false
d. A is false but R is true.
81. The Chola ruler who wrested
Tondaimandalam from the Rashtrakutas
was
a. Parantaka I
b. Sundarachola
c. Rajaraja I
d. Kulottunga I
82. In the Cholla administrative system the
Sabha was
a. The village court of justice
b. An assembly of adult men in agraharas
c. A committee of people from all castes
for the management of temple
d. A committee of nattars Temple
83. The famous temple of Kailasa, hewn out
of the rock, is at
a. Elephanta
b. Ellora
c. Badami
d. Mahabalipuram
84. Match List I with list II and select the
correct answer.
List-I
A. Pratiharas
B. Chaulukyas
C. Chahamanas
D. Paramaras
List-II
1. Suvamagiri
2. Chandravati
3. Anahailapataka
4. Kanyakubja
5. Kalyani
Codes: A B C D
a. 2 3 4 1
b. 4 3 1 2
c. 1 2 4 5
d. 3 5 2 4
85. Which one of the following was known as
Manigramam
a. The quarters for jewelers
b. The principal seat of the worship of
Yaksh Manibhadra
c. An association of traders
d. Villages granted tax tree to Brahmins
in the Chola
86. The name of Ekanta Ramayya is
associated with
a. Srivaishnavism
b. Kalamukha sect
c. Virasaivism
d. Jainism
87. Grant of land to the Buddhist monastery
Sailendra Chudamani vihara was made by
a. Pala king Devapala
b. Chola king Kulottunga
c. Pandyaq king Sundra Pandya
d. Pallava king Narasimha Varman
88. Which one of the following regions is
known for paintings that re manuscript
illustrations in miniature executed on
palm-leaf during the period A.D. 1100-
13000?
a. Kerala
b. Deccan
c. The Chola kingdom
d. Western Bihar
89. Somapura, the site of a great Buddhist
monastery was located in.
a. Eastern Bihar
9 of 11
b. Northern Bihar
c. Northern Bengal
d. Southern Bengal
90. Dahala in the early medieval period was
the territory of the
a. Paramaras
b. Chandellas
c. Kalachuris
d. Bhanjas
91. In the Delhi Sultanate one silver tanker
was the equivalent of
a. 52 jitals
b. 48 jitals
c. 45 jitals
d. 38 jitals
92. “I issue such orders to the people as I
consider to be beneficial for them and the
State. I do not know whether they are
permitted by the shariat or not “These was
said by
a. Jalaludding Khalji
b. Alaudding Khalji
c. Ghiasudding tughlaq
d. Muhammad Tughlaq
93. Given below are tow statements, one
labeled as Assertion (A) and the other
labeled as Reason (R):
Assertion (A): The lqta system went a
long way in establishing Islam in north
India.
Reason (R): the sultans, through the
institution, wanted to control the Turkish
nobles.
In the context of the above two statements,
which one of the following is correct.
a. Both A and R are true and R is the
correct explanation of A
b. Both A and R are true but R is not a
correct explanation of A
c. A is true but R is false
d. A is false but R is true.
94. Match List I with list II and select the
correct answer.
List-I: Years
A. 1296 A.D.
B. 1336 A.D.
C. 1398 A.D.
D. 1504 A.D.
List-II: Event
1. The Mongols invaded Delhi.
2. Harihara founded the kingdomof
Vijayanagara
3. Timur’s invasion
4. The city of Agra founded by sultan
Sikandar Lodi.
5. Transfer of the capital to Daulatabad
from Delhi.
Codes: A B C D
a. 2 3 5 4
b. 4 2 1 3
c. 3 1 5 4
d. 1 2 3 4
95. The main feature that distinguished the
Saqiya (Persian Wheel from other waterlifting
services during the DelhiSultanate
was the use of
a. Lever- principle
b. Pulleys
c. Gear mechanism
d. Leather bucket
96. During the thirteenth century the term
fawazil denoted
a. The surplus revenue remitted to the
State
b. A headman of a pargana
c. A military rank
d. The crown land
97. What was the total Jama of the delhi
Sultanate as estimated by Husamudding
Junaidi in the reign of Sultan Feroze Shah
Tughlaq?
a. Four crore and fifty lakhs
b. Five crore and thirty lakhs
c. Six crore and seventy – five lakhs
d. Seven crore and forty lakhs
98. The change of the capital of the bahamani
Kingdom from Gulbarga to Bidar took
place during the reign of
a. Shihabudding Ahmad I
b. Ghiyasudding Tahamtan
c. Shamsudding Daud
d. Tajuddin Firoz
99. Which one of the following pairs is not
correctly matched?
a. Baz Bahadur : Malwa
b. Sultan Muzaffar Shah : Gujarat
10 of 11
c. Yusuf Adil Shah : Ahmednagar
d. Qutb Shah : Golkunda
100. The central administration of the
Vijayanagara kingdom maintained contact
with the village through and officer called
a. Naik
b. Mahanayakacharyya
c. Mahapratihara
d. dandapasika
101. Match List I with list II and select the
correct answer.
List-I:
A. Appayya Dikshita
B. Kasivilasa Kriya Sakti
C. Vedanta Desika
D. Vyasaraya
List-II
1. Dwaita’
2. Adwaita
3. Kalamukha
4. Visishtadwaita
5. Kapalika
Codes: A B C D
a. 2 3 5 4
b. 4 2 1 3
c. 3 4 2 5
d. 2 3 4 1
102. To which one of the following sufi Orders
did Shaikh bahaudding Zakaria ofMultan
belong?
a. Chishti
b. Suhrawardi
c. Qadiri
d. Naqshbandi
103. Who among the following was/were in the
First Jesuit Mission that came to Akbar’s
Court?
1. Francis Xavier
2. Rudolph Aquaviva
3. Antony monserrate
4. St. Augustine
a. 1 and 2
b. 2 and 3
c. 1 and 4
d. 2 only
104. The founder of the Roshnal sect was
a. Abdullh Jaunpuri’baqi billah
b. Bayazid
c. Sheikh Nurul Haq
105. The Rajput clan to which Jhujhar singh
(who rebelled against the Mughals during
Shah Jahan’s reign) belonged was
a. Rathor
b. Sisodia
c. Chauhan
d. bundela 
106. Shahjahan’s rebellion in 1622 was
connected with Emperor Jahangir’s order
to
a. go to Bengal as Governor
b. surrender Prince Khusrau
c. attack Malik Ambar
d. lead an expedition to Qandahar
107. In the Mughal administration, Muhtasib
was
a. A military officer
b. An eccliciastical officer
c. An officer in charge of public morals
d. The head of the department of
correspondence
108. Who among the following Mughal rulers
introduced the doaspah sih-aspath rank in
the mansab system?
a. Akbar
b. Kajamgor
c. Sjag Kajam
d. Aurangzeb
109. According to the estimate of W.H.
Moreland the population of Indiain A.D.
1600 was.
a. 80 million
b. 100 million
c. 120 million
d. 125 million
110. The three methods of land revenue
assessment-batai, khetbatai and lang batai
were related to
a. Ghallabakshi
b. Nasaq
c. Kankut
d. Zabt
111. The number of the zamindar’s retainers in
Akbar’s reign according to Abul Fazl was
a. About 44 lakhs
b. About 48 lakhs
11 of 11
c. About 52 lakhs
d. About 55 lakhs
112. The court painter sent by Jahangir along
with his envoy to the Shag of Persia was
a. Bishandas
b. Abul Hasan
c. Mansur
d. Goverdhan
113. Who among the following was the painter
who was given the title nadir-uz-Zaman by
Jahangir?
a. Abul Hasan
b. Mansur
c. Bichiti
d. bishandas
114. Match List I with list II and select the
correct answer.
List-I: Royal Personages
A. Babur
B. Prince Khusrau
C. Nur Jahan
D. Rabia-ud-Dauran
List-II: Tombs at
1. Lahore
2. Kabul
3. Allahabad
4. Aurangabad
Codes: A B C D
a. 2 3 1 4
b. 2 1 4 3
c. 4 2 3 1
d. 1 4 2 3
115. The term pictra dura refers to
a. Wall painting
b. Inlay work
c. Sculptures in Italian style
d. A special type of turrets
116. The Razmnama is a Persian translation of
the
a. Panchtantra
b. Ramayana
c. Mahabharata
d. Kathasaritsagara
117. Which of the following is/are correctly
matched?
1. Portugal : Estado da India
2. France : Vereenigde Ost-Indische
Compagnic
3. Holland : Gcompagnic des Indes
Orientales
a. 1 only
b. 1 and 2
c. 1 and 3
d. 2 and 3
118. The Mughal emperor who issued a fireman
to the English East India company in 1717
to trade freely in Bengal was
a. Bahadur Shah
b. Shah Alam
c. Farrukh Siyar
d. Jahandir Shah
119. Match List I with list II and select the
correct answer.
List-I: Posts in the Maratha
Administration
A. Peshwa
B. Panditrao
C. Pant Amatya
D. Samant
List-II: Functions
1. Foreign affairs
2. Audit and accounts
3. Providing grants to scholars
4. General supervision
5. Military affairs
Codes: A B C D
a. 2 3 4 5
b. 4 1 2 3
c. 4 1 2 3
d. 3 1 4 2
120. Which of the following is true regarding
the treaty Purandar?
1. The treaty was concluded in June 1665
2. The treaty was concluded after the
official coronation of Shivaji.
3. As a result of the treaty Sambhaji was
granted the mansab of Five Thousand.
4. As a result of the treaty Shivaji handed
over 23 forts the Mughals.
5. As a result of the treaty Shivaji
Accepted the Mughal suzerainty.
a. 1 and 2
b. 1, 2, 3 and 4
c. 1, 3 4
d. 3, 4 and 5
 

IAS Prelims History/ Quiz

 1. Match List I with list II and select the
correct answer by using the codes given
below the lists:
List -I
A. Ajanta
B. Ellora
C. Khajuraho
D. Elephants
List-II
1. Temple built by the Chandela rulers
2. Cave paintings
3. Kailasha temple
4. Brahamanical sculptures
Codes:
A B C D
a. 2 3 1 4
b. 2 4 1 3
c. 1 3 4 2
d. 3 2 1 4
2. Match List I with list II and select the
correct answer by using the codes given
below the lists:
List -I
A. Rudramal Temple
B. Kandareya mahadeva Temple
C. Udayesvara Temple
D. Lingaraja Temple
List-II
1. Khajuraho
2. Bhuvaneswar
3. kanchipuram
4. Siddhapur
5. Udaipur
Codes:
A B C D
a. 3 1 4 2
b. 4 5 3 1
c. 4 1 5 2
d. 3 4 5 2
3. Who among the following had referred to
the lost glory and plendour of Pataliputra?
a. Cosmas
b. Fa-hien
c. Huien Tsang
d. Megasthenes
4. Who was the provincial ‘Governor of the
area marked ‘X’ at the time of
Chandragupta Maurya?
a. Menander
b. Pushyagupta
c. Tushaspha
d. Rudradaman
5. The earliest inscription in Sanskrit is found
at
a. Sanchi
b. Girnar
c. Lumbini
d. Sarnath
6. Before the Ghorian conquest muslim rulers
were found in
a. Oudha
b. Katehar
c. Sind
d. Delhi
7. Evidence regarding Chola paintings has
been found at
a. Uraiyur
b. Puhar
c. Thanjavur
d. Kanchipuram
8. What is the correct chronological order in
which the following Chola kings ruled?
1. Rajaraja III
2. Rajendra III
3. Rajadhiraja II
4. Vikrama Chola
Choose the correct answer using the codes
given below.
a. 4, 3, 2, 1
b. 3, 4, 2, 1
c. 3, 4, 1, 2
d. 4, 3, 1, 2
9. Match List I with list II and select the
correct answer by using the codes given
below the lists:
List –I (Deccan State)
A. Bidar
B. Ahmadnagar
C. Bijapur
D. goleunda
List-II (Founder)
1. yusuf Adil Shah
2. Qutub Shah
3. Amir Ali Barid
4. Malik Ahmad
Codes:
A B C D
a. 1 2 3 4
b. 4 3 2 1
c. 3 2 1 4
d. 3 4 1 2
10. Which one of the following sultans
brought the Ashokan Pillar toDelhi?
a. Ghiasudding Tughlaq
b. Firuz Tughlaq
c. Alauddin Khalji
d. Muhammad Tughlaq
11. The Delhi Sultan who established a
ministry of Agricultural development was
a. Balban
b. Muhammad Tughlaq
c. Alaudding Khalji
d. Firuz Tughlaq
12. Which of the following measures were
taken by Balban for re-establishing the
prestige and power of the Crown?
1. He claimed his descent from the
mythical hero Afrasiyab.
2. He copied the court etiquettes of the
Iranian Court.
3. He instituted rigid formalities in the
harem.
4. He reorganized the boundaries of
provinces of the kingdom.
Select the correct answerer using the codes
given below:
a. 2 and 3
b. 1 and2
c. 3 and4
d. 2 and4
13. In the Delhi Sultanate, the highest rural
authority for land venue was
a. Chaudhari
b. Rawat
c. Rana
d. Malik
14. The fixation of land revenue on the basis
of measurement of land and estimation of
yield per biswa was done first under.
a. Alauddin Khalji
b. Muhammad Tughalaq
c. Firuz Shah tughlaq
d. Sikandar Lodi
15. Amir khusrau composed his Nuhsiphir in
Praise of
a. Alaudding Khalji
b. India
c. Khwaja Moinuddin Chishti
d. Prophet of Islam
16. The Vijayanagar Emperor who made the
greatest contribution to Telugu and
Sanskrit literature was
a. Devaraya I
b. Devaraya III
c. Krishnadevaraya
d. Ramaraya
17. The Vijaynagara ruler who tried to
strengthen his miliniary position by
recruiting in his army 10,000 Muslim
mounted bowmen, was
a. Bukk I
b. Devraya II
c. Virupaksha Raya
d. Vir Narsimha
18. The chronological order in which the
following revelers, viz
1. nicolo conti
2. Abdur Razzaq
3. Ibn Batuta and
4. Barbosa
19. Visited India is:
a. 1, 3, 4, 2
b. 3, 1, 2, 4
3 of 12
c. 3, 1, 4, 2
d. 1, 3, 2, 4
20. Which one of the following medieval
Bhakti saints was patronized by Raja
Lakshman Sent of Bengal as one of the
‘Fife Gems of his Court?
a. Vallabhacharya
b. Jaidev
c. Chitanya
d. Narsi Mehta
21. According to Moreland, the jama signified
a. total revenue assessment
b. total land revenue assessment
c. land revenue realized
d. net income of the ruling class
22. In the Mughal Empire, the land revenue
was stated mainly in
a. Copper dams
b. Solver rupees
c. Gold muhrs
d. Maunds
23. “No one should interfere with the peoples
religious beliefs, for one deliberately
chooses an incorrect religion.”
This view was expressed in
a. Advice to Bughra Khan
b. Advice to Humyun
c. Letter to Shah Abbas
d. Message to Khusrau
24. Which of the following accounted for
agrarian crisis during late seventeenth
century?
1. Inbuilt weaknesses in jajir system
2. Monetary fluctuations
3. Natural calamities
4. Administrative failures
Choose the correct answer by using the
codes given below:
a. 1 and 2
b. 3 and 4
c. 1, 2 and3
d. 2, 3 and 4
25. Which one of the following sets of officers
was not found in the Provincial
Administration of the Mughals?
a. Subehdars, Faujdars, Karoris
b. Faujdars, Diwan, Bakhshi
c. Muhtasib, Wazir, Amil
d. Wazir, Bakhshi, Mir Saman
26. Which one of the following pairs in not
correctly matched?
a. 1560 …….. Fall of Bairam Khan
b. 1564……… Conquest of Gondwana
c. 1570………Fall of Ranthambor
d. 1576………Battle of Haldighati
27. The first English Captain to disembark at
Surat and who also visited the Court of
Emperor Jahangir at Agra was
a. Thomas Stephens
b. Hawkings
c. Ralph fitch
d. John Mildenhall
28. Match List I with list II and select the
correct answer by using the codes given
below the lists:
List –I(Period)
A. 1645 AD
B. 1637 AD
C. 1630 AD
D. 1666 AD
List-II (Event)
1. Aurangzeb was appointed Viceroy
Deccan
2. Gujarat Famine
3. Transfer of capital to Shahjahanabad
4. Death of Shahjahan
5. Capture of Kangra fort
Codes:
A B C D
a. 3 1 5 4
b. 3 1 2 4
c. 4 3 1 2
d. 1 3 2 4
29. Which one of the following statements is
not correct?
a. The vakil was a prestigious office in
rank and dignity
b. Akbar separated the revenue and
financial powers from office of vakil.
c. Office of vakil assumed importance
during the later part of Aurangzed’s
reign.
d. Jahangir ruled without a vakil for a
long time.
30. Which one of the following pairs is not
correctly matched
a. Arif Quandhari : Tarikh-i-Akbar
4 of 12
b. Nizamudding Ahamad: Tabaqat-i-
Akbari
c. Abdul quadir Badaoni : Muntakhab-ut
Tawarikh
d. Mutamad Khan: Khulastut 
31. Khawaja Mansur, the court painter of
Jahangir was a celebrated painter of
a. Portraits of birds and animals
b. Portraits of Mughal nobles
c. Landscapes
d. frescoes
32. Khwaja Ahdus Samad was the court
painter of
a. Babur
b. Akabar
c. Jhangir
d. Shahjahan
33. The ‘Fatwai Almgiri’ complied at the
order of Aurangzeb in 1663 by six persons
headed by Shaik Nizam was a
a. Book of administrative rules
b. Set of military regulations
c. Set religious regulations
d. Collection of sayings of Aurangzeb
34. Saikh Ahmad Sirhindi was imprisoned by
a. Humayun
b. Akbar
c. Jahangir
d. Shahjahan
35. The statement the “both Agraand
Fatehpur Sikri are bigger thanLondon”
was made by
a. Bernier
b. Ralph Fitch
c. Hawkins
d. Manucci
36. The author of the Das Badh written during
the Maratha period was.
a. Dadaji Konddeva
b. Ramdas
c. Tukaram
d. Eknath
37. Who among the following sons of
Aurangzeb sought the help of Marathas
against his father?
a. Akbar
b. Muazzam
c. Azam
d. Kambakhsha
38. Match List I with list II and select the
correct answer by using the codes given
below the lists:
List–I (Officials in Shivaji’s
administration)
A. Amtya
B. Dandadhyiksha
C. Sachiv
D. Sumant
List-II (Designation)
1. Ecclesiastical head
2. Home Secretary
3. foreign Secretary
4. Finance Minister
Codes:
A B C D
a. 1 4 3 2
b. 4 1 2 3
c. 4 1 3 2
d. 1 4 2 3
39. As a part of dowry on the marriage of
Princess Catherine of Braganza with
Charles II Bombay was given to the
British by the
a. French
b. Danes
c. Spaniards
d. Dutch
40. During the 17th Century, the trade between
Indian and Java was dominated by the
a. Portuguese
b. Dutch
c. English
d. French
41. In the given map, the largest part of the
Mughal Empire in the 17th century was the
one shown at
a. A
b. B
c. C
d. D
5 of 12
42. The effects of Revenue Reforms
introduced by Murshid Quli Khan in
Bengal raised the annual revenue to nearly
a. Eighty lakh rupees
b. One and half crore rupees
c. One crore rupees
d. Two crore rupees
43. Which of the following territories were
restored to the Mughal Emperr Shag Alam
II by the English?
a. 24 Paraganas
b. Kara and Allahabad
c. Bihar and Orissa
d. Awadh and Orissa
44. Which one of the following statements
correctly defines the term ‘Hindu Pad-
Padshahi’?
a. Maratha Swarajya
b. Maratha Empire
c. Maratha Imperialism
d. Hindu Swarajya
45. In the succession conflict after Ranjit
Singh’s demise, power was usurped by
a. Dhaian Singh
b. Nao Nihal Singh
c. Sher Singh
d. Kharag Singh
46. Consider the following statements about
Tipu Sultan
1. He made no treaty with the British.
2. He had not quarrel with the Marathas.
3. He sought French alliance.
4. He fell fighting against the British.
Of these statements
a. 1, 2 and are 3 Correct
b. 2, 3 and 4 are correct
c. 3 and 4 are correct
d. 1 and 4 are correct
47. Raushan Akhtar ascended the throne with
the title:
a. Jahandar Shah
b. Shah Jahan II
c. Mohammad Shah
d. Almgir II
48. What was the main cause of conflict
between Mir Quasim and the East India
Company?
a. Mir Quasim’s alliance with the Mughal
Emperor and the Nawab of Avadh
b. Abuse of Dastaks by the English
c. Mir Qasim’s conspiracy with the
French
d. Mir Jajar’s conspiracy with the East
India company
49. Siraj-ud-daula lost his Masnad because of
a. His weak army
b. His religious policy
c. Widespread economic distress
d. Intrigues and treachery at home
50. What is the correct chronological order of
the following British officials inIndia?
1. sir John Shore
2. marquis Cornwallis
3. Earl of Minto
4. Marquess of Wellesley
Select the correct answer using the codes
given below:
a. 2, 1, 4, 3
b. 1, 2, 3, 4
c. 1, 2, 4, 3
d. 2, 1, 3, 4
51. Which one of the following statements
correctly describes the Dual Government
arrangement made by the East India
Company in Bengal?
a. Executive and Judicial authority were
separated
b. Revenue collection was entrusted to
the deputy Nawab while military
affairs were retained by the Company.
c. Revenue administration and the
administration of criminal justice were
separated and were with the British
and the Nawab respectively
d. The new arrangement was made to
encourage agriculture
52. The economic policy of the British in India
was to
a. Industrialize India
b. Bring India to the level of Western
countries
c. Promote the material prosperity of the
Indian
d. Convert India into an agrarian satellite
of metropolitan
53. Which one of the following Acts abolished
the trading of the East India Company?
a. Regulating Act of 1773
b. Charter Act of 1813
6 of 12
c. Charter Act of 1833
d. Charter Act of 1853
54. Under the Permanent Settlement, the
Zamindar’s share in the collected land
revenue was
a. 1/4
b. 1/3
c. 1/11
d. 1/6.
55. Foreign capitalists were attracted by Indian
industry for a variety of reasons. Which
one of the following reasons was not one
of them?
a. Labour was extremely cheap
b. Aw materials were readily and cheaply
available
c. Indian capitalist class was well
developed but unable to compete.
d. For many Indian products there was a
ready demand the world over
56. Who among the following was not a
member of the congress Socialist Party,
although he encouraged and blessed its
formation?.
a. Achyuta Patwardhan
b. Jawaharlal Nehru
c. Jayaprakash Narayan
d. Acharya Narendra Dev
57. Rani Gaidilliu, a rebel leader against the
British, was from
a. Tripura
b. Assam
c. Nagaland
d. Manipur
58. The author who described the Revolt of
1857 as the first War of Independence was
a. Ashok Mehta
b. R.C. Jajumdar
c. S.N. Sen
d. V.D. Savarkar
59. the Window Remarriage Act was passed
during the Governor- Generalship of
a. Bentick
b. Dalhousie
c. Canning
d. Lawrence
60. Match List I with list II and select the
correct answer by using the codes given
below the lists:
List–I
A. Raja Rammohan Roy
B. Keshab Chandra Sen
C. Dayanand Saraswati
D. Ramakrishna Paramahansa
List-II
1. Observed that Brahamoism should
become a sort of universal religion.
2. Identified Hinduism with the religion
as founded in the Vedas
3. Emphasized that there are Many roads
to God.
4. Proclaimed that the purest form of
Hindu religion was to be found in
Upanishads.
Codes:
A B C D
a. 1 4 2 3
b. 1 4 3 2
c. 4 1 3 2
d. 4 1 2 3
61. The cultural background of Revolutionary
Terrorism in Bengal had been strengthened
by the writings of
a. Michael Madhusudhan Dutt
b. Sakharam Ganesh Deuskar
c. Bankim Chandra Chattopadhyay
d. Rabindranath Tagore
62. The lower castes tried to assets themselves
by borrowing and imitating customs and
manners of the upper castes. This was
termed as Sanskritisation by
a. Sadipada Banerjee
b. N.M. Lokhande
c. M.N. Roy
d. S.M. Joshi
63. The lower castes tried to assets themselves
by Borrowing and imitating customs and
manners of the upper cast This was termed
as Sanskritisation by
a. M.KI. Gandhi
b. Bernard Cohn
c. B.r. Ambedkar
d. M.N. Srinivas
64. In the elections of 1937, the Muslim
League’s electoral performances in the
Muslim majority provinces such as Sindh,
Punjab and North West Frontier had been.
a. Extremely poor
7 of 12
b. Very good
c. Good
d. Very poor
65. Subhas Chandra Bose fought the Congress
Presidential elections in 1939 on the
ground that
a. He did not like the nomination of
Sitaramayya
b. He pleaded for elections instead of
nominations
c. He was opposed to Gandhiji
d. He favoured an aggressive policy
which was opposed by the Congress
leaders.
66. During the later half of the nineteenth
century, the Indian intelligentsia started
protesting against the policies of the
colonial rule by various means. Which one
of the following was not a part of the
protest?
a. Writing books and articles on the
subject
b. Publishing newpaers for articulating
their views
c. Forming societies, Associations and
organizations to voice their grievances
d. Permanent Settlement
67. Lord Macaulay was associated with
a. Reforms in the army
b. Abolition of Sati
c. Codification of laws
d. Permanent Settlement
68. The President of the Indian National
Congress at the time of Indian
independence was
a. Maulana Abul Kalam Azad
b. Jawaharlal Nehru
c. Dr. Rahjendra Prasad
d. J.B.Kripalani
69. Rabindranth Tagore gave up his
knighthood because of
a. Jallianwala Bagh tragedy
b. Brutal suppression of Civil
disobedience Movement
c. Execution of Bhagat Singh
d. Chauri chaura incident
70. Match List I with list II and select the
correct answer by using the codes given
below the lists:
List–I
A. A.O. Hume
B. Sir Syaed Ahmad Khan
C. Minto
D. Fuller
List-II
1. ‘Hindus and Muslims were two eyes of
India’
2. Enacted Seditious meeting act
3. Lt. Governor of East Bengal and
Assam
4. Secretary of Indian national congress
Codes:
A B C D
a. 4 1 3 2
b. 1 3 2 4
c. 4 2 1 3
d. 4 1 2 4
71. modernization of Muslims inIndia was
initiated by
a. Sir Syed Ahmad Khan
b. Abdul latif
c. Badrudding Tayabji
d. Syed Amir Ali
72. Which one of the following pairs is not
correctly matfched
a. Deoband School.. Saiyyad Ahamd
Madani
b. Majilis-i-Ahrar.. Maulana Azad
c. Khaksar.. Inayat Ullah Mashriqui
d. Khudai Khidmatgar.. Abdul Ghaffar
Khan
73. There was little sympathy in spite of loud
professions loyalty. Moderate and
extremist alike learnt with satisfaction of
German victories. There was not love for
Germany of course, only the desire to see
our rulers humbled. Who among the
following write this?
a. C. R. Das
b. Jawaharalal Nehru
c. Gandhi
d. Rabindranath Tagore
74. Which one of the following pairs is
correctly matched
1. Gokhle .. British Indian Association
2. Narayan … All India trade Union
Congress Malhar Joshi
3. Annie Besant.. Servants ofIndia
society
8 of 12
4. Dadabhai naoroji .. Home-Rule League
75. Assertion (A): Hinayana and Mahayana
Buddhism were divided into four
speculative Schoos, Vaibhashika,
Sautrantika, Madhymika and Yogachara.
Reason (R): References to these four
Schools are to be found in the Brahmanical
works on philosophy
a. Both A and R are true and R is the
correct explanation of a.
b. Both A and R are true but R is Not the
correct explanation of A.
c. A is true but R is false
d. A is false but R is true. 
76. Assertion (A): The Indo-Aryans came to
India from somewhere in Central Asia,
possibly from the region south of the Urals
and east of the Caspian Sea.
Reason (R): they had been defeated by
their Indo-European neighbours.
a. Both A and R are true and R is the
correct explanation of a.
b. Both A and R are true but R is Not the
correct explanation of A.
c. A is true but R is false
d. A is false but R is true.
77. Assertion (A) The Bhagavata religion
spread to western India and north Deccan.
Reason (R): The Yadava-Stvata- Vrishni
people of Mathura migrated to different
regions.
a. Both A and R are true and R is the
correct explanation of a.
b. Both A and R are true but R is Not the
correct explanation of A.
c. A is true but R is false
d. A is false but R is true.
78. Assertion (A) Patanjali attests that during
the Mauryan times, new cults and super
stations were introduced for the sake of
money
Reason (R): The treasury needed to be
adequately filled to meet any unforeseen
economic crisis in the kingdom.
a. Both A and R are true and R is the
correct explanation of a.
b. Both A and R are true but R is Not the
correct explanation of A.
c. A is true but R is false
d. A is false but R is true.
79. Assertion (A): The Kushanas carried on
seat through the Persian Gulf and the Sea.
Reason (R): They maintained a wellorganized
navy.
a. Both A and R are true and R is the
correct explanation of a.
b. Both A and R are true but R is Not the
correct explanation of A.
c. A is true but R is false
d. A is false but R is true.
80. Assertion (A) The post-Gupta agrarian
structure marked by growing feudal set-up
Reason (R) It was the emergence of
land intermediaries and subjected
peasantry on large scale.
a. Both A and R are true and R is the
correct explanation of a.
b. Both A and R are true but R is Not the
correct explanation of A.
c. A is true but R is false
d. A is false but R is true.
81. Assertion (A): Some Pallava grants
prohibited the entry of royal officers in the
granted Village.
Reason (R): The granted villages were
directly administered by the king.
a. Both A and R are true and R is the
correct explanation of a.
b. Both A and R are true but R is Not the
correct explanation of A.
c. A is true but R is false
d. A is false but R is true.
82. Assertion (A): Anuloma marriages were in
vogue during the Gupta period.
Reason (R): The Gupta monarches wanted
to absorb the foreigners into the Hindu
fold.
a. Both A and R are true and R is the
correct explanation of a.
b. Both A and R are true but R is Not the
correct explanation of A.
c. A is true but R is false
d. A is false but R is true.
83. Assertion (A): Pushyamitra has signaled
his success by performing the Asvamedha.
Reason (R): Vasumitra had influcted a
defeat on the Yavanas.
a. Both A and R are true and R is the
correct explanation of a.
9 of 12
b. Both A and R are true but R is Not the
correct explanation of A.
c. A is true but R is false
d. A is false but R is true.
84. Assertion (A): The Chola ruler Rajendra’s
atrocities during his conquest ofSri Lanka
have been highlighted in the Mahavmsa.
Reason (R): The author had a strong
sympathy for Buddhism.
a. Both A and R are true and R is the
correct explanation of a.
b. Both A and R are true but R is Not the
correct explanation of A.
c. A is true but R is false
d. A is false but R is true.
85. Assertion (A): The Bhakti Saint
Vallabhacharya for professed Suddhavaita
philosophy
Reason (R): He completely identified the
Individual soul with Brahaman and did not
subscribe to the distinction between the
two.
a. Both A and R are true and R is the
correct explanation of a.
b. Both A and R are true but R is Not the
correct explanation of A.
c. A is true but R is false
d. A is false but R is true.
86. Assertion (A): The agrarian crisis began in
the later part of the 17th century.
Reason (R): the oppressed peasantry
joined the local reel leaders and zamindars.
a. Both A and R are true and R is the
correct explanation of a.
b. Both A and R are true but R is Not the
correct explanation of A.
c. A is true but R is false
d. A is false but R is true.
87. Assertion (A): the Buland Darwaza at
Fatehpur Sikri was built by Akbar in 1602.
Reason (R): he wanted to commemorate
his conquest of Gujarat.
a. Both A and R are true and R is the
correct explanation of a.
b. Both A and R are true but R is Not the
correct explanation of A.
c. A is true but R is false
d. A is false but R is true.
88. Assertion (A): Sulhkul, namely, peace
with all became the state policy of Akbar.
Reason (R): he removed religious
disabilities of the Hindus by abolishing the
pilgrim taxes, Jiziyah and opening state
service to them.
a. Both A and R are true and R is the
correct explanation of a.
b. Both A and R are true but R is Not the
correct explanation of A.
c. A is true but R is false
d. A is false but R is true.
89. Assertion (A): Jaisingh succeeded in
persuading Shivaji to come to Aurangzeb’s
court and enter Imperial serfices.
Reason (R): The attempt at reconciliation
failed because Shivaji was dissatisfied
with the terms offered.
a. Both A and R are true and R is the
correct explanation of a.
b. Both A and R are true but R is Not the
correct explanation of A.
c. A is true but R is false
d. A is false but R is true.
90. Assertion (A): Soon after the resignation
of the congress ministries in the provinces
in 1939, the Muslim League observed a
Deliverance Day’.
Reason (R) Ambedkar supported and
jointed it.
a. Both A and R are true and R is the
correct explanation of a.
b. Both A and R are true but R is Not the
correct explanation of A.
c. A is true but R is false
d. A is false but R is true.
91. At which one of the following sets of
Harappan sites, have structures with fire
altars been discovered?
a. Chanhudaro and Amri
b. Banwali and kalibangan
c. Harappan and Moohenjodaro
d. Almgirpur and Daimbad
92. A bronze chariot of Harappan times yoked
to a pair of bullock and driven by nude
human figure has been discovered at.
a. Sanghol
b. Rojdi
c. Kunal
d. Daimabad
10 of 12
93. A highly advanced water management
system of Harappan times has been
unearthed at
a. It is distinctly proto-Dravidian
b. It is distinctly Sanskriti
c. It is not alphabetical but mainly
pictographic
d. The language used by these scripts has
definite Sumerian connections.
94. Which one of the following is more
probable regarding the Harappan script?
a. It is distinctly proto-Dravidian
b. It is distinctly Sanskritic
c. It is not alphabetical but mainly
pictographic
d. The language used by these scripts has
definite Sumerian connections
95. Ananthroupomorphic figure of copper has
been reported from the Harppan levels at
a. Lothal
b. Kalibagan
c. Harppa
d. Rojdi
96. ‘Brahm’ emerged as the Supreme entity
during the times of the
a. Rigveda
b. Brahmanas
c. Aranyakas
d. Upanishads
97. the material culture of the Later Vedic
people in the Ganga Yamuna valley may
be gleaned from
a. Ochre-coloured pottery
b. Painted grey ware
c. Black and red ware
d. Northern Black polished ware
98. Match List (Sutra Literature) with List II
(Subject they deal with) and select the
correct answer by using the codes given be
low the lists:
List–I
A. Kalpa
B. Siksha
C. Nirukta
D. Chhanda
List-II
1. Etymology
2. Phonetics
3. ritual
4. Metre
Codes:
A B C D
a. 4 1 2 3
b. 1 4 3 2
c. 4 1 3 2
d. 1 4 2 3
99. Match List I with list II and select the
correct answer by using the codes given
below the lists:
List–I
A. Tirukkural
B. Silppadikaram
C. Manimekalai
D. Tolkappiyam
List-II
1. Ilango Adigal
2. Sathanar
3. Tolkappiyar
4. Tiruvalluvar
Codes:
A B C D
a. 4 1 2 3
b. 1 4 3 2
c. 4 1 3 2
d. 1 4 2 3
100. The earliest available work of the Sangam
Tamils is
a. Pattinappalai
b. Tirumurugarruppadai
c. Maduraikanchi
d. Tolkappiyam
101. Which one of the following factors was
most responsible for the origin of
Buddhism?
a. Change of tribal state into territorial
state
b. Impact of the increasing use of iron
c. Growth of the guild system
d. Challenge of the kshtriyas to the
dominance of the brahamanas.
102. Which one of the following Buddhist sects
insists on the necessity of the monastic
life, worship of the relics of Lord Buddha,
attainment of salvation by oneself alone
and in the belief that Lord Buddha will no
more be born?
a. Hinayana
b. Mahayana
11 of 12
c. Vajrayna
d. Lamaism
103. The Jain temple known for the finest
marble carvings in India are situated at
a. Gwalior
b. Kiradu
c. Modhera
d. Dilwara
104. Which of the following kings of ancient
Indian were patrons of Jainis?
1. Senika Bimbisara
2. Chandragupta Maurya
3. Kharvela
Choose the correct answerer using the
codes given below:
105. Match List I with list II and select the
correct answer.
List–I
A. Balarama
B. Rama Dasarathi
C. Kalki
D. Vamana
List-II
1. Battle axe
2. Dwarf
3. bow and arrow
4. Horse and sword
Codes:
A B C D
a. 1 2 4 3
b. 2 3 1 4
c. 1 3 4 2
d. 3 1 4 2 
106. Match List I with list II and select the
correct answer.
List–I
A. Visvanatha
B. Vasudeva
C. Minakshi
D. Eknamsa or Bhadra
List-II
1. Puti
2. madurai
3. Kasi
4. Mathura
Codes:
A B C D
a. 2 1 3 4
b. 3 1 2 4
c. 3 4 2 1
d. 2 3 4 1
107. Which of the following formed the basis of
the materials culture in the Maurya period?
1. Intensive use of iron
2. Abundance of punch-marked coins.
3. Plenty of NBPW.
4. Use of burnt bricks.
5. Use of ring-wells
Select the correct answer using the codes
given below:
a. 1, 2, 3, and 5
b. 1, 3 and 4
c. 2, 4 and 5
d. 1, 2, 4 and 5
108. Which one of the following pairs is not
correctly matched?
a. Fourteen major rock Edicts: Various
Principles of Dhamma
b. Seven Pillar Rock Edicts: Appendix to
Rock Edicts
c. Bhabru Inscription: New
Administration by Asoka
d. Barabar Cave Inscription: Ashok’s
Tolerance.
109. During the Gupta period the term
Bhagakara meant
a. Land tax
b. Sales tax
c. Tax on wastelands
d. Tax on mines
110. Which one of the following can be inferred
from the paucity of coins in the later Gupta
period?
a. Decline of trade
b. Decline of money economy
c. Non-availability of gold
d. Self sufficient village economy
111. Match List I with List II and select the
correct answerer by using the codes given
below the lists:
List–I
A. Setthi
B. Sreni
C. Nikkha
D. Kamsa
List-II
1. Gold coin
12 of 12
2. Bronze and copper coin
3. Chief of a guild
4. Guild
Codes:
A B C D
a. 3 4 2 1
b. 4 3 1 2
c. 3 4 1 2
d. 4 3 2 1
112. Which one of the following statements is
not true of the post-Gupta agrarian
structure?
a. It was marked by the emergence of
landed intermediaries.
b. It saw the growth of subject peasantry
c. It was rooted in the absolute royal
ownership of land
d. There were grown evidences of
agrarian expansion
113. Which of the following are marked by the
agrarian structure in the post-Gupta
period?
1. Large scale grants of land indifferent
part of India
2. Undermining of communal rights over
land.
3. Subjection of peasants.
Select the correct answer using the codes
given below:
Codes:
a. 1 and 2
b. 2 and 3
c. 1 and 3
d. 1, 2 and 3
114. Which of the following statements explain
the aspect feudalism in Ancient India?
1. The king granted lands to his officers
in lieu of salaries.
2. The work of cultivation was carried
out by sudra peasants
3. some of the powerful feudatories had
their own sub-feudatories.
Select the correct answer using the codes
given below:
Codes:
a. 1, 2 and 3
b. 1 and 2
c. 2 and 3
d. 1 and 3
115. In the early medieval period the system of
land grants did not exist i
a. Orissa
b. Uttar Pradesh
c. Punjab
d. Rajsthan
116. The place at which the first Sati pillar
dated 510 A.D was found is
a. Earn
b. Chotegaon
c. Lothal
d. Dangvada
117. Which one of the following ancient
Smritikaras gave the window a position
next to that of the son in the matter of
inheritance?
a. Yajnavalkya
b. Manu
c. Brhspati
d. Katyayana
118. To which category can works such as
‘Jyotisha edanga’ and ‘Surya-Prajnapti’ be
assigned?
a. Cosmology
b. Astrology
c. Astronomy
d. Horscopy
119. Which one of the following places is
known for its monumental temple
architecture?
a. Arikkamedu
b. Amaravati
c. Gangaikondacholapuram
d. Uttiramerur
120. Which one of the following Chaityas is
circular?
a. Bhaja
b. Bidsa
c. Junnar
d. Karle

IAS Prelims History/ Quiz

 1. Who among the following had written a

treatise on fanciful technological devices?
a. Bhoja
b. Govindaraja
c. Chandravarman
d. Mahipala
2. Which one of the following communities
of peasants revolted against the Pala rulers
of Bengal?
a. Jats
b. Gurjaras
c. Kavartas
d. Doms
3. A very important source for the political
structure and social conditions ofGujarat
from the 9th to the 13th centuries is the text
known as
a. Rajatarangini
b. Panchatantra
c. Lekhapadhati
d. Sukraniti
4. The Chola inscriptions are found in the
following languages:
1. Sanskrit
2. Telugu
3. Tamil
4. Kannada
Their correct sequence in terms of the
extent to which they were used (most
frequent first, least frequent last) is
a. 1, 3, 4, 2
b. 3, 1, 4, 2
c. 1, 3, 2, 4
d. 3, 1, 2, 4
5. Match the temples in List built by the
Chola monarchs with the places in List II
and select the correct answer:
List-I
A. The Koranganatha Temple

B. The Rajarajeswara Temple
C. The Airavatesvara Temple
D. The Shival Temple
List-II
1. Tiruvaliswaram
2. Dharasuram
3. Srinivasanallur
4. Tanjore
A B C D
a. 3 4 1 2
b. 3 4 2 1
c. 4 3 2 1
d. 4 3 1 2
6. The capital – seat Gangaikonda
Cholapuram was founded by
a. Rajendra I
b. Rajaraja the Great
c. Kulottunga I
d. Parantaka
7. Which one of the following pairs is
correctly matched?
a. Krishnadevaraya : Manucharita
b. Bukka I: Vaidikamanggapravartaka
c. Allasaniqeddanna: Amuktamalyada
d. Gangadevi: Amarsataka
8. Abdur Razzsaq who left is detaled account
of Vijayanagara Empire in Persian, came
to the Vijayanagar court as the envoy of
Mirza Shag Rukh during the reign of
a. Harihara II
b. Bukka II
c. Deva Raya II
d. Krishnadeva Raya
9. What was the nature of the Nagaram
settlements South India?
a. Villages inhabited by peasant castes
b. Villages settled and dominated by
combinations of traders and merchants
c. Villages granted to the Brahamanas
d. Villages having the local
administrative units
10. The territorial revenue assignment in the
Delhi Sultanate was known as
a. Tuyul
b. Milk
c. Iqta
d. Jagir
11. Sultan Alaudding Khalji claimed land
revenue on
a. One-fourth of he produce
b. Two-thirds of the produce
c. One-half of the produce
d. Two-fifths of the produce
C.S.E. Pre-1999
INDIAN HISTORY
2 of 12
12. Which one of the following was the reason
for the great rebellion in Doab against
Muhammad Tughlaq?
a. Transfer of capital from Delhi
b. Issue of token currency
c. Increase in rural taxation
d. Enslavement of rural population
13. Yahiya Sirhindi, the author of Tarikh-i-
Mubarakshahi live during the period of
a. Lodis
b. Soyidds
c. Tughulqs
d. Khalijis
14. Match List I with List II and select the
correct answer:
List-I
A. Silver tanka
B. Diwan-i-Kohi
C. Dar-ul-Shafa
D. Shahan-i-Mandi
List-II
1. Alaudding Kahalji
2. Feroze Tughluq
3. Iltutmish
4. Muhammad Tughluq
A B C D
a. 4 3 2 1
b. 3 4 1 2
c. 3 4 2 1
d. 4 3 1 2
15. ‘When he attained kingship, he was quite
independent of the rules and orders of the
shariat.’
Ziauddin Barani made the above statement
regarding
a. Iltutamish
b. Alauddin Khalji
c. Mubarak Khalji
d. Muhammad Tughluq

16. Match List I with List II and select the

correct answer:
List-I (Author)
A. Minhaj Siraj
B. Amir Khusrau
C. Hasan Nizami
D. Shams-i-Siraj Afif
List-II (Works)
1. Tarikh-e-firuz Shahi
2. Tajul Ma’asir
3. Khazainul Futuh
4. Tabaquat-i-Nasiri
A B C D
a. 3 4 2 1
b. 3 4 1 2
c. 4 3 1 2
d. 4 3 2 1
17. The tomb of Ghiyasuddin Tughluq is
marked by
a. a bulbous dome
b. sloping walls
c. the absence of marble
d. high towers
18. Who among the following were prominent
members of the Nirguna cult of Bhakti
Marge?
1. Kabir
2. Nanak
3. Raidas
4. Mira
Choose the correct answer using the codes
given below:
a. 2 and 3
b. 1, 3 and 3
c. 1, 3 and 4
d. 1, 2, 3 and 4
19. Which of the following places were
captured by Krishnadevaraya from
Prataparudra?
1. Raichur
2. Udayagiri
3. Penukonda
4. Kondapalli
Choose the correct answer using the codes
given below
a. 1, 2 and 4
b. 2 and 3
c. 1 and 3
d. 2 and 4
20. Which one of he following Sufi saints saw
the rule of seven Sultans but visited the
Court of none?
a. Qutbudding Bakhtiyar Kaki
b. Baba Farid-ud-din Chishti
c. Shaikh Nizamudding Chishti
d. Nasirudding ‘Chiragh’ Delhi
21. the longest canal of its time which is still
used and is most beneficial to North India
was built by
a. Sher Shah Suri
b. Babur
c. Firuz Shah
d. Firuz Shah Shahajahan
3 of 12
22. Manucci says that the re-imposition of
Juziya was strongly opposed by a group
of nobles headed by
a. Jaswant Singh
b. Jahanara
c. Rana Raj Singh
d. Shayista Khan
23. Match List I with List II and select the
correct answer
List-I (Events
A. Conquest of Malwa
B. Formation of Subas
C. Annexation of Kashmir
D. Establishment of the Department of
News Reporting
List-II (Years)
1. 1562
2. 1574-75
3. 1580
4. 1586
A B C D
a. 3 1 2 4
b. 1 3 2 4
c. 3 1 4 2
d. 1 3 4 2
24. ‘Amongst his great nobles he (the king)
was so predominant that none dared lift his
head too high, but with the humbler
classes he was benevolent and debonair,
willingly giving them audience and
hearing their petitions. He was pleased to
accept their presents, taking them into his
hands and holding them to his beast.’
Consider the quotation given above and
indicate who it is attributed to and who
was the king referred to in it.
a. Monserrate : Aakabar
b. Tavernier : Jahangir
c. Fitch : Shahajahan
d. Hawkins: Aurangzeb
25. Who among the following rulers was very
proficient in dancing and music?
a. Adil Shah Sur
b. Babur
c. Akbar
d. Jahangir
26. In the Mughal Empire, the salaries of the
nobles fixed.
a. Arbitrarily in each case by the
Emperor
b. According to their mansab number
c. One the basis of the size of their
contingent
d. On the basis of their race and
competence
27. Which of the following were the main
features of the imperial Mughal land
revenue system?
1. It varied according to the crop grown
2. It varied from region to region
3. It varied according to farming bids
received
4. It varied according to individual
contracts with village headmen.
Select the correct answer using the codes
given below:
a. 2,3 and 4
b. 1 and 4
c. 1 and 2
d. 1 and 3
28. Abul Fazal justifies the realization of
revenues by Akbar as it was the
a. Remuneration of sovereignty
b. Divine right of the king
c. Cost of maintaining the army for the
protection borders
d. Hereditary proprietary right of the king
29. The Satnamis who rebelled in the time of
Aurangzeb and occupied the town of
Narnaul, were followers of
a. Kabir
b. Dadu
c. Guru Nanak
d. Ravidas
30. ‘So complete was the ruin that not a cat or
dog was left among the buildings of the
city, in its place or its suburbs.,
The above statement about the transfer of
capital by Sultan Muhammad Bin Tughlaq
was made by
a. Amir khusrau
b. Isami
c. Ibn Batuta
d. Ziaudding Barani
31. Consider the following statements:
In the 1560’s Akbar began to patronize the
formation of the Mughal Schoolof
painting with the illustration of Datan-i-
Amir Hamza, because.
1. Akbar inherited Humyun’s taste for
painting.
2. Akbar was especially interested in
book illustration .
4 of 12
3. Akbar was persuaded to take interest in
painting by Abul Fazl.
Of these statements
a. 1, 2 and 3 are correct
b. 2 and 3 are correct
c. 1 and 2 are correct
d. 1 and 3 are correct
32. Match List I with List II and select the
correct answer:
List-I (Monuments)
A. Taj Mahal
B. Fatehpur Sikri palace complex
C. Gold Gumbad
D. Sher Shah’s tomb
List-II (Features)
1. Extensive use of trabeate forms
2. Largest dome built in pre-modern India
3. surrounded by water
4. Bulbous dome
A B C D
a. 1 4 3 2
b. 4 1 3 2
c. 1 4 2 3
d. 4 1 2 3
33. The founding Charter of the English East
India Company was granted by
a. Henry VIII
b. Elizabeth I
c. James I
d. Charles I
34. The first Danish settlement of trade at
Tranquebar was established in the year
a. 1620
b. 1630
c. 1660
d. 1616
35. Marathi chronicles were known as
a. Khyat
b. Bakhar
c. Burunji
d. Bahi
36. During the second Mratha invasion of
Bengal, at the following came to support
the Bengal Nawab against Raghjuji
Bhonsle’s forces?
a. Peshwa Balaji Baji Rao
b. Nizamul Mulk
c. Holkar
d. Jai Singh Sawai
37. Match the places marked A,B, C and D on
the given map with the descriptions given:
1. Capital of he Sharqi kingdom
2. Important centre of production of
indigo
3. Site of first true dome
4. Capital of Adil Shahis
5. The place to which Muhammad
Tughluq transferred his capital
A B C D
a. 4 1 3 2
b. 2 3 4 1
c. 2 1 3 5
d. 4 3 1 2
38. The founder of the autonomous kingdom
of Avadh was
a. Ahmed Shah Abdali
b. Safdarjung
c. Saadat Khan Burhan-ul-Mulk
d. Zulfiqar Khan
39. A modern arsenal was established in 1765
with French assistance at
a. Bidnur
b. Sera
c. Sunda
d. Dindigul
40. Who planted the Tree of Liberty’ at
Srirangapatnam?
a. Hyder Ali
b. Chikka Krishna Raj
c. Tiput Sultan
d. Devraj
41. Who among the following rulers of the
Travancore kingdom extend his dominion,
organized a standing army and a rational
administration and encouraged foreign
trade in the middle of 18th century?
a. Rama Varma
b. Zamorin of Calicut
c. Martanda Verma
d. Nanjaraj
42. The jat State of Bharatpur was set up by
a. Churaman
b. Surajmal
5 of 12
c. Boi Singh
d. Sawai Raja jai singh
43. Which of the following are associated with
Ranjit Singh?
1. Sukerchakia misl and the capture of
Lahore
2. Battle of Ambala and the defeat of
Lord Lake
3. Signing subsidiary alliance withEast
India company
4. Employing Faqir Azizuddin and
Dinanath
a. 1 and 2
b. 2 and 3
c. 1 and 4
d. 3 and 4
44. Who was the woman leader of the
Marathas who carried on struggle against
the Mughal Empire from 1700 AD
onwards following the death of Raja Ram?
a. Ahalyabi
b. Miktabai
c. Tarabai
d. Rukminibai
45. The Maratha who helped the Sayyad
brothers in over throwing Farukh Siyar in
1719 was
a. Shivaji II
b. Sahuji
c. Balaji Vishwanath
d. Baji Rao I
46. What is the correct chronological sequence
of the following rulers of Bengal?
1. Suja-ud-din
2. Murshid quili Khan
3. Sarfaraz Khan
4. Alivardi Khan
47. Consider the following Britishers officers:
1. James Grant
2. John shore
3. Arren hastings
4. Maccaughten
Those well- known in connection with the
revenue settlement include
a. 1 and 2
b. 1, 2 and 3
c. 3 and 4
d. 2 and 3
48. consider the following statement: The
Diwani rights assured the East India
company the power to
1. levy tariffs
2. collect land revenue
3. administer civil justice
Of these statements
a. 1 alone is correct
b. 2 and 3 are correct
c. 1, 2 anf 3 are correct
d. 1 and 3 are correct
49. The supreme Court at Calcutta was
established by
a. North’s Regulating Act of 1773
b. Pitt’s India Act of 1784
c. Charter Act of 1793
d. Charter Act of 1813
50. Dayal Singh Majithia was associated with
1. newspaper
2. Education
3. Banking
4. British Army
Select the correct answer using the codes
given below:
a. 1, 2 and 4
b. 1, 2 and 3
c. 2, 3 and 4
d. 1, 3 and 4
51. Identify the British territories as on 1765,
in the given map
a. 1 and 3
b. 2 and 4
c. 2 and 3
d. 1 and 2
52. Devan Vehu Thampi of Travancore
organized the Revolt in
a. 1800
b. 1805
c. 1809
d. 1811
53. Match List I with List II and select the
correct answer:
List-I
A. Bhil uprising
B. Kittur uprising
6 of 12
C. Satara uprising
D. Gadkari uprising
List-II
1. 1841
2. 1844
3. 1818-31
4. 1824
A B C D
a. 4 3 1 2
b. 3 4 1 2
c. 3 4 2 1
d. 4 3 2 1
54. Which one of the following rebellions is
associated with Sido and Panho?
a. The Antha Rebillion, 1855
b. The Kol uprising, 1820-37
c. The Munda ebellion, 1899-1900
d. Orissa Zamindars Rebellion, 1807-
1817
55. Match List ( Personalities connected with
the uprising of 1857) with List II ( Places)
and select the correct answer:
List-I
A. Kanwar singh
B. Maulavi Ahmadullah
C. Mangal pandey
D. Zeenat Mahal
List-II
1. Delhi
2. Bihar
3. Faizabad
4. Barrakpore
A B C D
a. 3 2 1 4
b. 2 3 1 4
c. 2 3 4 1
d. 3 2 4 1
56. Nizam-ul-Mulk established the
independent State of
a. Hasanpur
b. Hyderabad
c. Kandahar
d. Burhanpur
57. General Windham was defeated by the
rebel force 1857 near
a. Gwalior
b. Agra
c. Bhopal
d. Kanpur
58. The Asiatic Society of Bengal was
founded by
a. Ram Hohan Roy
b. Sir William Jones
c. W.W. Hunter
d. William Bentinck
59. Satyashodak Samaj founded by
a. Jotiba Phule
b. Sri narayan Guru
c. Gopal Babu Walong
d. Bhaskarrao jadav
60. the young Bengal Movement in the 19th
Century was inspired by
a. rasik Kumar Mallick
b. Ramtannu Lahari
c. Henry Vivian Derozia
d. Peary Chande Mitra
61. The Rajmundri Social Reform Association
to encourage window re-marriage was
founded in 1871 by
a. Virasalingam
b. K.T. Telang
c. Behramji
d. Gopalachariar
62. Match List I with List II and select the
correct answer:
List-I
A. Dayanand Sarswati
B. Lal Munshi Ram
C. Pandit Shiv Narayan Agnihotri
D. Lala Hansraj
List-I
1. Gurukul
2. Dev Samaj
3. Ango-Vedic Colleges
4. Satyarth prakash
A B C D
a. 1 4 2 3
b. 4 1 3 2
c. 4 1 2 3
d. 1 4 3 2
63. Match List I with List II and select the
correct answer.
List-I
A. Digdarshan
B. Mirat-ul-Akhbar
C. Statesman
D. Hindu patriot
List-II
1. Robert KInight
2. Harischanda mukherjee
3. J.C. Marshman
4. Ramohon Roy
7 of 12
A B C D
a. 3 4 2 1
b. 4 3 1 2
c. 3 4 1 2
d. 4 3 2 1
64. In which one of the following conspiracy
cases in India was an Englishman named
Mr. Philip Spratta tried?
a. Lahore Conspiracy case
b. Kanpur Conspiracy case
c. Meerut Conspiracy case
d. Chittagong Conspiracy case
65. Ahmedabad Txtile Labour Association
(ATLA) was founded by
a. Mridula Sarabhai
b. N.M. joshi
c. V.V. Giri
d. M.K. Gandhi
66. Match List I with List II and select the
correct answer.
List-I
A. Punnapara Vayalar movement
B. Tebhaga movement
C. Telengana moement
D. Eka movement
List-II
1. UP
2. Kerala
3. Bengal
4. Andhra
A B C D
a. 1 2 3 4
b. 2 1 4 3
c. 2 1 4 3
d. 1 2 4 3
67. Match the individuals given in List with
the movement given in List II and select
the correct answer:
List-I
A. Malati Choudhary
B. karuna Sindhu Roy
C. Bhagwan singh Longowalia
D. Yadunandan Sharma
List-II
1. Surma Valley no-rent struggle
2. Utkal Provincial Kisan SabhaPatiala
Muzara Movement
3. Bihar Kisan Sabha
A B C D
a. 1 2 3 4
b. 2 1 3 4
c. 2 1 4 3
d. 1 2 4 3
68. What is the correct chronological order of
the following movements of Gandhi?
1. Rowlatt Act aviation
2. Kheda movement
3. Champaran movement
4. Ahmedabad Mills strike
Select the correct answer using the codes
given below:
a. 3, 2, 4., 1
b. 3, 2, 1, 4
c. 2, 3, 1, 4
d. 2, 3, 4, 1
69. Har Dayal, Baba Harnam Singh, Tundilat’,
Babhakna’ and Gurdit Singh were
associated with
a. india Independence League
b. Nowjawan Bharat Sabha
c. Ghadar movement
d. akali movement
70. What is the correct chronological sequence
of the following events?
1. Guru-ka-high agitation
2. Vaikom Satyagraha
3. Kakori case
4. Nehru Report
Select the correct answer using the codes
given below:
a. 1, 2, 3, 4
b. 1, 2, 4, 3
c. 2, 1, 3, 4
d. 2, 1, 4, 3
71. ‘We have now an open conspiracy to free
the country from foreign rule and you.
comrades, and all our countrymen and
countrywomen are invited to join it.’
Who among the following made this
declaration?
a. M.N. Roy
b. Jayaprakash narayan
c. Subhash Bose
d. Jawaharlal Nehru
72. How many seats were given to the
‘depressed classes under the communal
Award and Poona Pact?
a. 74 and 79 respectively
b. 71 and 147 respective
c. 78 and 80 respectively
d. 78 and 69 respectively
8 of 12
73. Match List I with List II and select the
correct answer:
List-I
A. Sardual Singh Kaveeshar
B. Yusuf Meherally
C. T. Prakasam
D. Chowdhary Khaliquzaman
List-II
1. Muslim League
2. forward Block
3. Indian National Congress
4. Congress Socialist Party.
A B C D
a. 2 4 1 3
b. 4 2 1 3
c. 2 4 3 1
d. 4 2 3 1
74. Match List I with List II and select the
correct answer:
List-I (Works)
A. Cha man Athaguntha (Oriya, 1921)
B. Jivwanara Batat (Assamese, 1945)
C. Morali Mannige (Kannada, 1942)
D. Ranangan (marthi, 1939)
List-II (Authors)
1. Birinchin Kumar Barua
2. Fakir Mohan Senapati
3. visram Bedkar
4. Sivaram Karnth
A B C D
a. 1 2 3 4
b. 2 1 3 4
c. 2 1 4 3
d. 1 2 4 3
75. Match List I (personalities) with List II
(Special interests) and select the correct
answer:
List-I
A. Shambhu Mitra
B. Ram Kinkar
C. Ravi Varma
D. Pramathesh Barua
List-II
1. painting
2. Theatre
3. Films
4. Sculpture
A B C D
a. 2 4 1 3
b. 4 2 1 3
c. 2 4 3 1
d. 4 2 3 1

76. Match List (Events) with List II (Years )
and select the correct answer:
List-I
A. Founding of Federation of Indian
Cambers of Commerce and industry
(FICCI)
B. Lee-Modi Pact
C. Bombay Plan
D. National Planning Committee
List-II
1. 1938
2. 1927
3. 1944
4. 1934
A B C D
a. 4 2 3 1
b. 2 4 3 1
c. 2 4 1 3
d. 4 2 1 3
77. Assertion (A): Malik Amber was the first
to employ successfully and on a large scale
the Maratha light horse as guerillas.
Reason (R): He wished to weld the
Marathas in to a nation.
a. Both A and R are true and R is the
correct explanation of A
b. Both A and R are true but R is NOT
the correct explanation of A
c. A is true but R is false
d. A is false but R is true
78. Assertion (A): Akbar claimed divinity for
himself.
Reason (R): he though that in a country of
many beliefs such as India, the sovereign
should not be Associated with any one
particular religion.
a. Both A and R are true and R is the
correct explanation of A
b. Both A and R are true but R is NOT
the correct explanation of A
c. A is true but R is false
d. A is false but R is true
79. Assertion (A): the invasions of
Krishadevaraya into the territories of
Prataparudra were inconclusive.
Reason (R) Krishnadevaraya invaded the
territories of the Gajepathi not for the
latter’s extinction but only for the recovery
of lost territories.
a. Both A and R are true and R is the
correct explanation of a.
9 of 12
b. Both A and R are true but R is Not the
correct explanation of A.
c. A is true but R is false
d. A is false but R is true.
80. Assertion (A): Cottage industries and
handicrafts of India flourished due to the
development of railways.
Reason (R): Railways helped movement of
commodities and goods.
a. Both A and R are true and R is the
correct explanation of a.
b. Both A and R are true but R is Not the
correct explanation of A.
c. A is true but R is false
d. A is false but R is true.
81. Assertion (A): The arthashastra of
Kautilya provided for the office of the
superintendent of trade.
Reason (R) The State engaged in extensive
trade.
a. Both A and R are true and R is the
correct explanation of a.
b. Both A and R are true but R is Not the
correct explanation of A.
c. A is true but R is false
d. A is false but R is true.
82. Assertion (A): the Kushnas issued a large
number of gold coins.
Reason (R): the period was marked by
flourishing trade.
a. Both A and R are true and R is the
correct explanation of a.
b. Both A and R are true but R is Not the
correct explanation of A.
c. A is true but R is false
d. A is false but R is true.
83. Assertion (A): Urban centres in the
Gangetic plans declined during the Gupta
period.
Reason (R) Trade languished
a. Both A and R are true and R is the
correct explanation of a.
b. Both A and R are true but R is Not the
correct explanation of A.
c. A is true but R is false
d. A is false but R is true.
84. Assertion (A): Society became
increasingly agrarian during the post-
Gupta period.
Reason(R): Small peasants could not claim
any rights to land.
a. Both A and R are true and R is the
correct explanation of a.
b. Both A and R are true but R is Not the
correct explanation of A.
c. A is true but R is false
d. A is false but R is true.
85. Assertion (A) The post-Gupta period
witnessed the expansion of the Kayastha
caste in North India.
Reason (R): The Vaishyas gave up
agriculture completely.
a. Both A and R are true and R is the
correct explanation of a.
b. Both A and R are true but R is Not the
correct explanation of A.
c. A is true but R is false
d. A is false but R is true.
86. Assertion (A): The Shudras were largely
peasants during post-Gupta times.
Reason (R): The Vaishyas gave up
agriculture completely.
87. Which one of the following Harappan
Sites is NOT located in Gujarat?
a. Surkotada
b. Rangpur
c. Sutkagendor
d. Desalpur
88. Math List I with List II and select the
correct answer:
List-I
A. Mohenjodaro
B. Lothal
C. Banawali
D. Kalibangan
List-II
1. Twin-burials
2. Corbelled drain
3. Furrows
4. Terracotta replica of a plough
A B C D
a. 1 2 3 4
b. 2 1 4 3
c. 1 2 4 3
d. 2 1 3 4
89. Who among the following has worked on
the Decipherment of the Indus Script?
a. H.D. Sankalia
b. B.B.Lal
c. I. Mahadevan
d. Vincent Author Smith
90. the Copper hoards are associated with the
10 of 12
a. lustrous red ware
b. ochre-coloured ware black and red
ware
c. black and red ware
d. panted grey ware

91. Which one of the following sites contains
the evidence of cemetery H culture?
a. Chanhudaro
b. harappa
c. Hohenjodaro
d. Kalibangan
92. Which one of the following statements
regarding Purushasukta is incorrect?
a. it occurs in the seventh mandala of the
Rigveda
b. It mentions the Brahmana as
emanating from the mouth of the
Primeval Being.
c. It shows the rise of hierarchy
d. It accords religious sanction to the then
emerging social structure.
93. the Vaishya is characterized as a tributary
to another (anyasya balikrita) and to be
eaten or lived upon by another
(anyyasyadya) in the
a. Vajasaneyi Samhita
b. Aitareya Brahmana Atharvaveda
c. Atharvaveda
d. Shatapatha Brahmana
94. Which one of the following is NOT true of
woman the Rigveda?
a. They attended the proceedings of the
Sabha
b. They performed the sacrifice
c. They took an active part in battles
d. They were married before puberty
95. Along with the ‘Late Harappan’ artefact,
painted grey ware has been found at
a. Ropar
b. Bhagwanpura
c. Bhanawali
d. Daulatpur
96. Caste is NOT Characterized by
a. endogamy
b. division of labour
c. equity
d. heredity
97. The concept of Varnasamkara occurs first
in the
a. Brahamanas
b. Upanishads
c. dharmasutras
d. Smritis
98. The nirvasita (excluded) and aniravasita
(not excluded) shudras have been referred
to by
a. Yaska in the Nirukta
b. Panini in the Ashtadhyayio
c. kautilya in the Arthashastra
d. None of the above
99. Math List I with List II and select the
correct answer:
List-I
A. Kautilya
B. Manu
C. Yajnavalkya
D. Narada
List-II
1. The householder should not quarrel
with a slave
2. There are fifteen kinds of slave
3. No Arya can be a slave
4. Nobody can be enslaved without his
consent
A B C D
a. 1 3 4 2
b. 1 3 2 4
c. 3 1 4 2
d. 3 1 2 4
100. Which one of the following combinations
is true of the Rigvedic culture?
a. nature worship, varnasystem, image
worship
b. Rural, pastoral, monarchical
c. Barter system, sati system sacredness
of cow
d. matriarchy, monogamy, defensive
weapons
101. Which of the following characterized the
Tamill society of Sangam Age?
1. Complex caste system
2. Prohibition of intermarriage between
the kings and velalas
3. Vegetarianism among the Brahmanes
4. Gradual Aryanisation
Selection the correct answer using the
codes give below:
a. 1 alone
b. 1 and 2
c. 2 and 3
d. 4 alone
102. What is the correct chronological order in
which the following appeared in Tamil
land?
1. Damili Inscriptions
2. Sangam Age
3. Rock-cut Temples
Select the correct answer using the codes
given below:
a. 1, 2, 3
b. 2, 1, 3
c. 3, 1, 2
d. 3, 2, 1
103. The first translator of Mahabharata into
Tamil was
a. Perundevanar
b. Kamban
c. Sundaramurthi
d. Bharavi
104. Who among the following was the earliest
known Greek follower of Bhagwatism?
a. Demetrius
b. Antialkides
c. Heliodorus
d. Menander
105. Which one of the following pairs is NOT
correctly matched?
a. Udiyanjeral: chera king
b. Nedumjeliyan : pandya king
c. Senganan: chola king
d. Pari: Pallava king

106. The jain tex which contains the
Biographics of the Firthankaras is known
as
a. Bhagavati Sutra
b. Adi Purana
c. Kalpasutra
d. Uvasagadasao

107. Consider the following statements:
In the second Buddhist Council held at
Vaishali, Buddhism was divided into
1. Sthaviravadins
2. Mahasanghikas
3. Vajrayana
4. Kalachakrayana
Of these statements
a. 1, 2, 3 and 4 are correct
b. 1, 3 and 4 are correct
c. 2, 3 and 4 are correct
d. 1 and 2 are correct
108. The earliest epigraphical evidence
regarding hagavatism in Indiacomes from
a. Ghosundi Stone inscription of king
Sarvatata
b. Besanagar inscription of Heliodorus
c. Nanaghat cave inscription of the
Satvahana queen Naganika.
d. Mehrauli Pillar inscription of Chandra
109. That Bhagavatism first grew up around the
Yamuna in Mathura district is evident fro
the account of
a. Justin
b. Arian
c. Megasthenes
d. Strabo
110. Which one of the following is NOT true of
the punch marked coins?
a. They are the earliest extant coins in
India
b. They bear stamps of more than one
punch
c. Symbols of different types are
represented on them
d. They always bear inscriptions of the
issuers
111. Which one of the following objects of
Roman manufacture has been most
abundantly found in India?
a. Pot-sherds
b. Bronze icons
c. Lamps
d. Coins
112. Four sites have been labelled as A,B,C and
D in the given map
Inscription detailing Samudragupta’s
conquests in the South has been found on
an Ashokan Pillar at the site labelled
a. A
b. B
c. C
d. D
113. Match List I with List II and select the
correct answer:
List I (Type of spies)
A. Gudhapurushah
B. Samsthah
C. Sancharah
D. Rupajivah
12 of 12
List II (nature of their work)
1. courtesans
2. Wandering spies
3. Secret emissaries
4. Stationary spies
A B C D
a. 4 3 2 1
b. 3 4 1 2
c. 3 4 2 1
d. 4 3 1 2
114. Match List I with List II and select the
correct answer:
List I (Names of the territories)
A. Uttarapatha
B. Avantirattha
C. Sancharah
D. Rupajivah
List II (Their capitals)
1. Tosali
2. Pataliputra
3. Takshashila
4. Ujjayaini
A B C D
a. 4 3 1 2
b. 3 4 2 1
c. 4 3 2 1
d. 3 4 1 2
115. Match List I with List II and select the
correct answer:
List I (King who presented gifts to their
contemporaries)
A. Sandrocottus
B. Seleucus
C. Ambhi
D. Antiochus (I. Soter)
List II (Gifts)
1. 80 talents of stamped silver
2. A large portion of Ariana
3. Sweet wine and dried figs.
4. 500 elephants
A B C D
a. 2 4 1 3
b. 2 4 3 1
c. 4 2 1 3
d. 4 2 3 1
116. The medium of exchange prevalent during
the Mauryan periods was
a. Kapaddaka-purana
b. Churni
c. Pana
d. Dinara
117. What is the correct chronological sequence
of the following?
1. Geography of Ptolency
2. Ctesias account of India
3. Christian Topography of Cosmas
indicopllustes
4. Pliny’s natural History
a. 4, 2, 3, 1
b. 2, 4, 1, 3
c. 2, 4, 3, 1
d. 4, 2, 1, 3
118. In his account, Fa-hien does NOT refer to
a. Peasce and order in the Middle
kingdom
b. Severity of criminal justice
c. Chandalas as social outcastes
d. Vegetarianism of the common people
119. Which one of the following is NOT true of
the Rajputs?
a. Their rise was an important social
phenomenon of the early medieval
period.
b. They comprised the ruling and warrior
chiefs
c. They were a landed aristocracy
d. The Medas were denied entry to their
ranks
120. Which one of the following rulers granted
land to high official for their maintenance?
a. Bimbisara
b. Chandragupta Maurya
c. Ashoka
d. Harshavardhana

आईएएस (IAS) प्रारंभिक परीक्षा 2011 जीएस मॉडल पेपर-प्रथम प्रश्न-पत्र हल सहित: सेट I

आईएएस (IAS) प्रारंभिक परीक्षा 2011 के सामान्य अध्ययन प्रथम प्रश्न-पत्र की तैयारी में सहायता हेतु विविध विषयों से संबंधित प्रश्नों को यहां संकलित करके दिया गया है. परीक्षार्थी इन प्रश्नों के माध्यम से अपनी तैयारी को सही दिशा देकर सफलता प्राप्त कर सकते हैं.
1. कांग्रेस जनता का सूक्ष्म मात्र है मैं इसकी शांतिपूर्ण मौत में सहयोग दूंगा किसने कहा था?
(a) डफरीन        
(b) कर्जन       
(c) लैंसडाउन   
(d) लार्ड एल्गिन
Answer : (b) कर्जन
2. वर्ष 1896 में विवेकानंद द्वारा बनाया गया रामकृष्ण मिशन का मुख्य उद्देश्य क्या था?
(a) समाज सेवा   
(b) शून्य पर शोध   
(c) कृष्ण भक्ति   
(d) राम भक्ति
Answer : (a) समाज सेवा
3. बनारस में केन्द्रीय हिंदू विश्वविद्यालय की स्थापना किसने की?
(a) जानथन डक्कन       
(b) एनीबेसेट   
(c) मदनमोहन मालवीय       
(d) श्रद्धानंद
Answer : (c) मदनमोहन मालवीय
4. महिलाओं के आत्मा चेतना एवं आत्मविश्वास के लिए ऑल इंडिया वुमेंस कांफ्रेंस कब संपन्न हुआ?
(a) 1907        
(b) 1897        
(c) 1927   
(d) 1917
Answer : (c) 1927
5. तहरीर चौक  किस शहर में स्थित है?
(a) साना       
(b) कायरो   
(c) त्रिपोली   
(d) आशाबाद
Answer: (b) कायरो
6. गैलेक्सी टेब टेबलेट गुगल द्वारा एंड्रोइड ऑपरेटिंग सिस्टम किस कम्पनी द्वारा जारी किया गया?
(a) डेल       
(b) सैमसंग   
(c) एलजी       
(d) एसर
Answer: (b) सैमसंग
7. सूची I को सूची II  से सुमेलित कीजिए और सूचियों के नीचे दिए गए कूट का प्रयोग कर सही उत्तर चुनिए :
सूची I                  सूची II
A. विकीपीडिया       1. जैक डेरोसी
B. फेसबुक            2. जिम्मी वेल्स
C. टियुटर            3. मार्क जैक वर्ग
D. विकीलिक्स       4. जुलियांस आसांज
कूट :
         A     B    C    D
(a)    2     4    3    1
(b)    2     3    1    4
(c)    2     1    3    4
(d)   1     2    3    4
Answer: (a) 2 4 3 1
8. वर्ष 2010 का मैन ऑफ बुकर पुरस्कार किस व्यक्ति को दिया गया?
(a) मारियो वागेसि       
(b) हार्वड जैकबसन       
(c) ग्रांट माटिन       
(d) डेविड कैली
Answer: (b) हार्वड जैकबसन

9. किसने कहा मानव जाति के लिए एक धर्म, एक जाति, एक ईश्वर का नारा किसने दिया?

(a) नारायण गुरू 
(b) ज्योतिबाफुले       
(c) गांधी जी   
(d) अम्बेडकर
Answer : (b) ज्योतिबाफुले
10. बंगाल विभाजन के समय कौन सी घटना संपन्न हुई?
(a) स्वदेशी बैंक एवं बीमा    
(b) राष्ट्रीय शैक्षिक संस्थाएं   
(c) राष्ट्रीय कॉलेज   
(d) उपरोक्त सभी 
Answer : उपरोक्त सभी
11. बंगाल विभाजन पर किसने कहा, वर्तमान परिस्थितियों में प्रशासन चलाना असंभव बना देंगें?
(a) लाला लाजपत
(b) तिलक   
(c) अरविन्द घोष
(d) विपिन चंद्र पाल
Answer : (c) अरविन्द घोष
12. भारत में फैले सांप्रदायिक विचार धारा के लिए कौन जिम्मेदार थे?
नीचे दिए गए कूट का प्रयोग कर सही उत्तर चुनिए :
(i) स्वामी दयानंद
(ii) सरसैय्यद
(iii) अंग्रेज
(iv) तिलक
(v) ज्योतिबाफुले
कूट :
(a) 1,2,3   
(b) 1,2,3,4
(c) 1,2,3,5
(d) 2,3,4,5
Answer : (a) 1,2,3
13. अखिल भारतीय हिंदू महासभा का पहला अधिवेशन कब हुआ?
(a) वर्ष 1915
(b) वर्ष 1920       
(c) वर्ष 1922       
(d) वर्ष 1925
Answer : (a) वर्ष 1915
14. ग़दर पार्टी की पत्रिका ग़दर का पहला संस्करण किस भाषा में था?
(a) अंग्रेजी       
(b) उर्दू       
(c) हिन्दी   
(d) फ़ारसी
Answer :(b) उर्दू
15. एंटी हेल गन सिस्टम सेब फतस को सुरक्षा प्रदान करने के लिए यूएसए तकनीक पर भारत में सबसे पहले कहां उपयोग में लाया गया?
(a) शिमला       
(b) श्रीनगर       
(c) जम्मू       
(d) अनंतनाग
Answer : (a) शिमला
16. कौन सुमेलित नहीं है?
(a) जी-20 बैठक-टोरांटो   
(b) द्वितीय इब्सा बैठक-दिल्ली   
(c) परमाणु सुरक्षा-बैठक-वाशिंगटन
(d) 16 वां सम्मेलन-सार्क-थिम्पू
Answer : (b) द्वितीय इब्सा बैठक-दिल्ली
17. सूची I को सूची II  से सुमेलित कीजिए और सूचियों के नीचे दिए गए कूट का प्रयोग कर सही उत्तर चुनिए:
सूची I            सूची II
A. मेगी         1. फिलीपिंस
B. लैला         2. अरब सागर
C. फेट          3. बंगाल की खाड़ी
D. लैसी         4. ऑस्ट्रेलिया
कूट :
      A    B   C    D
(a) 1    2   3    4
(b) 2    1   3    4
(c) 3     2  1    4
(d) 1    3   2    4

Answer : (d) 1 3 2 4

18. सी-17 ग्लोब मास्टर 3 क्या है जिसे भारत यूएसए से खरीदने का प्रयास कर रहा है?
(a) मिसाइल सुरक्षा प्रणाली           
(b) परमाणु जल युद्धपोत
(c) मिलीटरी ट्रांसपोर्ट हवाई जहाज       
(d) जासूसी-तंत्र
Answer : (c) मिलीटरी ट्रांसपोर्ट हवाई जहाज
19. वर्ष 1990 के शुरूआती समय में भारत पर बाध्य कर्ज का भार भारतीय जीडीपी (GDP) का कितना प्रतिशत था?
(a) 2
(b) 12
(c) 22
(d) 32
Answer : (c) 22
20. भारत सरकार ने विदेशी निवेश में एफपीआई (F.P.I) की अनुमति कब से दी?
(a) वर्ष 1991       
(b) वर्ष 1992       
(c) वर्ष 1994   
(d) वर्ष 1999
Answer : (b) वर्ष 1992
21. प्रतिस्पर्धा आयोग (Competition commission of India)  का अध्यक्ष किसे बनाया जाता है? (सभी सेवानिवृत्त हैं ).
(a) सर्वोच्च न्यायालय के न्यायाधीश    
(b) उच्च न्यायालय के न्यायाधीश   
(c) रिजर्व बैंक ऑफ इंडिया के गवर्नर   
(d) सीएजी (CAG)
Answer : (a) सर्वोच्च न्यायालय के न्यायाधीश
22. लघु स्तरीय उद्योग एसएसआई (S.S.I) के क्षेत्र में हैण्डलूम को बढ़ावा देने के लिए सरकार कौन सी योजना चला रही है?
(a) दीन दयाल प्रोत्साहन योजना   
(b) राजीव  गांधी विकास योजना
(c) इंदिरा  गांधी हथकरघा योजना    
(d) नेहरु हथकरघा योजना
Answer : (a) दीन दयाल प्रोत्साहन योजना
23. स्विस बैंक में जमा विभिन्न देशों में किस देश का स्थान प्रथम है?
(a) रूस       
(b) यूएसए    
(c) चीन       
(d) भारत
Answer : (d) भारत
24. 18 वां अंतरराष्ट्रीय एड्स सम्मेलन का आयोजन जुलाई 2010 में कहां किया गया?
(a) नई दिल्ली   
(b) वियाना   
(c) अम्बुजा   
(d) होनोई
Answer : (c) अम्बुजा
25. कालबेलिया (kalbelia) नृत्य जिसे यूनेस्को 2010 के द्वारा मानव सभ्यता के लिए विरासत घोषित किया. कहां होता है?
(a) गुजरात       
(b) उड़ीसा       
(c) महाराष्ट्र       
(d) राजस्थान
Answer : (d) उड़ीसा
26. गोलान हाईट डिस्पयूट किन दो देश में विवाद का विषय बना हुआ है?
(a) इसराइल-लेबनान   
(b) इसराइल-तुर्की       
(c) इसराइल-सीरिया        
(d) इसराइल-मिस्र
Anser : (c) इसराइल-सीरिया
27. वर्तमान समय में कार्बन डाई ऑक्साईड गैस प्रति वर्ष कितने प्रतिशत से वातावरण में बढ़ रहा है?
(a) 1%       
(b) 0.5%   
(c)  2%
(d) 10%
Answer : (b) 0.5%

आईएएस (IAS) प्रारंभिक परीक्षा 2011 जीएस मॉडल पेपर-प्रथम प्रश्न-पत्र हल सहित: सेट II

1. विधवा विवाह के सम्बन्ध में पुस्तक सत्य प्रकाश के लेखक कौन हैं?
(a) दयानंद सरस्वती        
(b) परमहंस   
(c) करसनदास       
(d) अगकर 
Answer : (c) करसनदास
2. किस गवर्नर ने कहा बहादुरशाह की मृत्यु के बाद मुग़ल सम्राट की पदवी किसी को नहीं दी जाएगी?
(a) डल्हौजी   
(b) कैनिंग       
(c) हेस्टिंग       
(d) लार्ड लिटन
Answer : (a) डल्हौजी
3. बैरकपुर के मंगल पांडे को फांसी कब हुई थी?
(a) 29 मार्च 1857        
(b) 29 जनवरी 1857   
(c) 29 मार्च 1856        
(d) इनमे से कोई नहीं
Answer : (a) 29 मार्च 1857
4. अवध (लखनऊ) में 1857 के विद्रोह का नेतृत्व कर्ता कौन था?
(a) जीनत महल        
(b) हजरत महल   
(c) बिरजीस कादीर
(d) अहमदुल्ला
Answer : (b) हजरत महल

5. निम्न में कौन सुमेलित नहीं है?

(a) मजदूर किसान शान्ति संगठन- अरुणाराय
(b) परिवर्तन- अरविन्द केजरीवाल
(c) नवज्योति- किरण वेदी
(d) एमवी फाउंडेशन- हर्ष मंदर
Answer: (b) एमवी  फाउंडेशन-हर्ष मंदर
6.संयुक्त राष्ट्र द्वारा शीर्ष (Top 10) दस विश्वविद्यालयों में भारत का एकमात्र विश्वविद्यालय कौन सा है? वर्ष (2010 में).
(a) जवाहर लाल नेहरु विश्वविद्यालय   
(b) दिल्ली विश्वविद्यालय         
(c) कोलकाता विश्वविद्यालय   
(d) अन्नामलाई
Answer : (c) कोलकाता विश्वविद्यालय
7. किस देश ने क्यूटो प्रोटो कॉल सम्मेलन 2010 में दूसरी बार अनुमोदित करने से इंकार कर दिया?
(a) रूस           
(b) जर्मनी       
(c) जापान       
(d) अन्नामलाई
Answer : (c) जापान
8. विश्व के वैश्विक धरोहर (रॉयल कासबी मकबरा ) जो आग से पूरी तरह नष्ट हो गया . कहां था?
(a) मोरक्को        
(b) लीबिया   
(c) युंगाडा   
(d) मिस्र
Answer : (c) युंगाडा
9. लार्ड रिपन के कार्यकाल में कौन से कार्य नहीं हुए?
नीचे दिए गए कूट का प्रयोग कर सही उत्तर चुनिए :
(1) स्थानीय स्वाशासन     
(2) प्रथम जनगणना   
(3) हंटर आयोग
(4) इल्बर्ट विधेयक    
(5) श्वेत विद्रोह
(6) कुका आंदोलन
कूट :
(a) 1,2,3,4,6      (b) 1,2,3,4       
(c) 1,2,3,4,5       (d) 1,2,3,5,6
Answer :  (c) 1,2,3,4,5
10. किस गवर्नर को प्रेस के मुक्तिदाता की संज्ञा दी जाती है?
(a) रिपन           
(b) मेटकॉफ     
(c) विलियम बैंटिक        
(d) लार्ड ऑकलैंड
Answer : (b) मेटकॉफ
11. पहली कपड़ा मिल 1853 में कावस जी नाना भाई ने कहां शुरू की?
(a) मुम्बई           
(b) अहमदाबाद    
(c) सूरत
(d) कलकत्ता
Answer : (c) सूरत
12. दीनबंधु मित्र ने नील दर्पण बंगला नाटक कब लिखा?
(a) 1860       
(b) 1870
(c) 1880
(d) 1890
Answer : (b) 1870
13. सूची I को सूची II से सुमेलित कीजिए और सूचियों के नीचे दिए गए कूट का प्रयोग कर सही उत्तर चुनिए:
            सूची I                           सूची II
A. ईस्ट इंडिया एसोसिएशन      1. फिरोजशाह मेहता
B. इंडियन एसोसिएशन           2. दादा भाई नरौजी
C. पूना सार्वजनिक सभा          3. सुरेन्द्रनाथ बनर्जी
D. मद्रास महाजन सभा           4. रघवा चार्य
Answer : 2,3,1,4
14. भारतीय कांग्रेस ने वर्ष 1890 में कौन सी पत्रिका निकाली?
(a) इंडिया       
(b) भारत        
(c) हिन्दुस्तान       
(d) आर्यावत्
Answer : (a) इंडिया
15. एआर रहमान ऑस्कर 2011 में किस फिल्म में संगीत देने के लिए नामांकित किए गए थे?
(a) द किंग्स स्पीच    
(b) 127 ऑवर्स
(c) द सोशल नेटवर्क
(d) इन्सेप्सन 
Answer : (b) 127 ऑवर्स
16. विश्व के शीर्ष आउटसोर्सिंग कंपनियों में शीर्ष 10 कंपनियों में भारत की कौन-2 सी कम्पनियां है?
नीचे दिए गए कूट का प्रयोग कर सही उत्तर चुनिए :
(1) इम्फोसिस  (2) 127 ऑवर   (3) विप्रो    (4) जेनपैक्ट    (5) टेक महिंद्रा
कूट :
(a) 1,2,3,4    (b) 1,2,3,5    (c) 1,2,3    (d) 1,2,3,4,5
Answer : (a) 1,2,3,4
17. 13 वें वित्त आयोग के अनुसार निम्न कथनों पर विचार कीजिए :
(1) केन्द्रीय करों की प्राप्तियों में राज्य का हिस्सा 32% है.
(2) केन्द्र को कुल प्राप्त राजस्व का 39% हिस्सा राज्य को देगा.
उपर्युक्त कथनों में से कौन सा/से सही है/हैं ?
(a) 1       
(b) 2       
(c) 1 और 2       
(d) ना तो 1 और ना ही 2
Answer : (c) 1 और 2
18. भारतीय रुपए को प्रतीक चिन्ह किसने दिया?
(a) डी उदय कुमार       
(b) टी प्रभात कुमार       
(c) एम जानकी प्रसाद   
(d) कुछ नहीं
Answer : (a) डी उदय कुमार
19. प्रथम हरित क्रांति के दूसरे चरण को किस क्षेत्र में लागू किया गया?
(a) दक्षिण एवं पूर्वी भारत में       
(b) दक्षिण-पश्चिमी भारत में       
(c) उत्तरी एवं पूर्वी भारत में       
(d) पूर्वी एवं पश्चिमी भारत में
Answer : (a) दक्षिण एवं पूर्वी भारत में
20. भारत में द्वितीय हरित क्रांति की शुरुआत सरकार ने कब से शुरू की?
(a) फरवरी 2004
(b) जून 2004   
(c) फरवरी 2005   
(d) अप्रैल 2005
Answer : (a) फरवरी 2004
21. बागवानी क्षेत्र के लिए मूल्य स्थिरता कोष का मुख्य उद्देश्य कृषकों को संकट के समय मूल्य स्थिरता की गारंटी देना है. यह कितने वर्षों के लिए है?
(a) 10 वर्ष       
(b) 5 वर्ष   
(c) 20 वर्ष   
(d) 15 वर्ष
Answer : (a) 10 वर्ष
22. भारत में पहला ईपीजेड (EPZ) का निर्माण कहां किया गया?
(a) कांडला   
(b) सूरत
(c) नवी मुंबई        
(d) नोएडा
Answer : (a) कांडला
23.  सूची I को सूची II से सुमेलित कीजिए और सूचियों के नीचे दिए गये कूट का प्रयोग कर? सही उत्तर चुनिए:
           सूची I                                    सूची II
         समझौता                                     देश
A. जैतापुर परमाणु आपूर्ति                       1. रूस
B. सैन्य असैन्य 30 समझौता                   2. ब्रिटेन
C. छह समझौता                                   3. चीन
D. अगले 5वर्षों में द्विपक्षीय व्यापार दुगुना      4. फ्रांस
कूट :
         A    B    C    D
(a)    4    1    3    2
(b)    4    2    3    1
(c)    2    1    3    4
(d)    1    2    3    4
Answer : (a) 4 2 3 1
24. किस देश की राजधानी की स्थापना का 1000 वां वर्षगांठ अक्टूबर 2010 में मनाया गया?
(a) फ्रांस   
(b) वियतनाम   
(c) दक्षिण अफ्रीका   
(d) जर्मनी
Answer : (b) वियतनाम
25. भारत ने किस तारीख को मतदाता दिवस के रूप में मनाने का निर्णय लिया?
(a) 25 जनवरी   
(b) 25 फरवरी   
(c) 25 जून
(d) 25 दिसंबर
Answer : (a) 25 जनवरी
26. राष्ट्रीय विज्ञान दिवस कब मनाया जाता है?
(a) 10 नवंबर       
(b) 28 फरवरी   
(c) 15 मार्च   
(d) 20 सितंबर
Answer : (b) 28 फरवरी
27. क्योटो प्रोटोकॉल के अनुसार औद्योगिक देश वर्ष 2012 तक कार्बन उत्सर्जन में  1990 की तुलना में कितना प्रतिशत कमी लाने का निर्णय लिया?
(a) 4%       
(b) 5%       
(c) 6%       
(d) 77%
Answer : (b) 5%
28. भारत में कुल कार्बन उत्सर्जन में कोयला (जीवाश्म ईंधन) आधारित बिजलीघरों का योगदान क्या है?
(a) 40%       
(b) 50%       
(c) 60%       
(d) 70%
Answer : (c) 60%
29. मीथेन गैस का उत्सर्जन निम्न में से किस स्थान से नहीं होता है?
(a) दलदली क्षेत्र       
(b) जुगाली करने वाले पशु       
(c) धान की खेती       
(d) वन आग
Answer : (d) वन आग
30. पूरे विश्व में प्रतिव्यक्ति कार्बन डाई ऑक्साइड का उत्सर्जन प्रतिवर्ष कितना है? (टन में).
(a) 2.5       
(b) 4.5       
(c) 6.5       
(d) 8.5
Answer : (b) 4.5
31. सूची I को सूची II से सुमेलित कीजिए और सूचियों के नीचे दिए गए कूट का प्रयोग कर सही उत्तर चुनिए :
    सूची I                           सूची II
A. कार्बन डाई ऑक्साइड         1. ओजोन लेयर टूटना
B. कार्बन मोनो ऑक्साइड        2. अम्लीय वर्षा
C. सल्फर डाई ऑक्साइड        3. सांस फेफड़ों समस्या
D. क्लोरोफ्लोरोकार्बन            4. तापवृद्धि
कूट :
         A    B    C    D
(a)    1    2    3    4
(b)    4    3    2    1
(c)    4    3    1    2
(d)    4    1    3    2
Answer : (b) 4 3 2 1
32. रिजर्व बैंक ऑफ इंडिया (RBI) के दिशा निर्देश के अनुसार उस धन को एनपीए (N.P.A) की संज्ञा दी जाती है. जिसका मूल ब्याज दर पीआईआर (P.I.R) निम्न दिनों के भीतर भुगतान नहीं किया जा सकता?
(a) 45 दिन        
(b) 90 दिन   
(c) 180 दिन   
(d) 360 दिन
Answer : (b) 90 दिन
33. कमजोर बैंकों के पुनर्संरचना के लिए गठित कौन सी समिति ने कमजोर बैंकों को मजबूत बैंकों के साथ विलय करने की घोषणा की?
(a) वर्मा समिति    
(b) नरसिंहमन समिति       
(c) केलकर समिति   
(d) इनमें से कोई नहीं
Answer : (a) वर्मा समिति
34. निम्नलिखित कथनों पर विचार कीजिए :
1. वैसे उद्योग जो पिछले 5 वर्ष से अस्तित्व में है तथा पिछले तीन वर्षों से लगातार घाटा में चल रहा है बीमार उद्योग कहलाते हैं?
2. वैसे उद्योग जो अपने कुल बाज़ार मूल्य का 50% तक ह्रास कर चुकी हैं कमजोर उद्योग कहलाती है?
उपर्युक्त कथनों में से कौन सा/से सही है/हैं ?
(a) 1, 2 सही     
(b) 1 सही
(c) 2  सही   
(d) ना तो 1 और ना ही 2
Answer : (a) 1, 2 सही
35. औद्योगिक रुग्णता को समाप्त करने के लिए कौन-कौन सी समितियां बनी है? नीचे दिए गए कूट का प्रयोग कर सही उत्तर चुनिए :
(1) ओंकार गोस्वामी   
(2) बालकृष्ण इराडी       
(3) आबिद हुसैन
कूट :
(a) 1, 2     (b) 2, 3    (c) 1, 3     (d) 1, 2, 3 
Answer : (a) 1, 2
36. भारत में एफडीआई (F.D.I) को तीव्र रूप से भारत में लाने के लिए भारत सरकार ने कौन सी समिति  वर्ष 2001 में बनाई?
(a) एनके सिंह        
(b) बीके गुप्ता   
(c) पीके सेन       
(d) सीके गुप्ता 
Answer : (a) एनके सिंह
37. भारत विश्व को किस का निर्यात सबसे अधिक करता है?
(a) पेट्रोलियम उत्पाद   
(b) इंजीनियरी सामान
(c) रसायन   
(d) रत्न भूषण 
Answer : (b) इंजीनियरी सामान
38. भारत में पेट्रोलियम उत्पाद के अतिरिक्त दूसरा सबसे अधिक किस वस्तु का आयात करता है?
(a) मशीनरी   
(b) खाद्य तेल   
(c) सोना एवं चांदी   
(d) लौह इस्पात
Answer : (a) मशीनरी
39. डब्ल्यूटीओ (W.T.O) में कृषि से सम्बंधित प्रमुख तीन स्तम्भ का भाग है कौन तीन स्तम्भ में सम्मिलित नहीं है?
(a) बाजार पहुंच   
(b) घरेलू सहायता   
(c) निर्यात प्रतियोगिता   
(d) आयात नीति
Answer : (d) आयात नीति
40. वर्तमान समय में डब्ल्यूटीओ (W.T.O) के कितने सदस्य देश हैं?
(a) 149   
(b) 151   
(c) 153   
(d) 155
Answer : (c) 153
41. सेज (S.E.Z)  द्वारा निर्यात को अधिक बढ़ावा दिया जा रहा है. वर्ष 2007-2008 के आधार पर वर्ष 2008-09 में सेज (S.E.Z) द्वारा निर्यात के क्षेत्र में कितने प्रतिशत वृद्धि हुई?
(a) 25
(b) 50        
(c) 75
(d) 90
Answer : (d) 90
42. भारत का अफ्रीकी देशों के समूह के साथ लगातार व्यापार बढ़ रहा  है. किस अफ्रीकी देश से सर्वाधिक व्यापार होता है?
(a) दक्षिण अफ्रीका     
(b) नाइजीरिया
(c) घाना   
(d) जिम्बाब्वे
Answer : (a) दक्षिण अफ्रीका
43. भारत की सबसे बड़ी व्यापार कम्पनी जो विदेशों के साथ व्यापार में संलग्न है?
(a) एमएमटीसी         
(b) एमएनटीसी
(c) एमएलटीसी    
(d) एमपीटीसी 
Answer : (a) एमएमटीसी
44. वायदा कारोबार देखने की मुख्य जिम्मेदारी वायदा व्यापार आयोग का है. इसका गठन कब किया गया है?
(a) वर्ष 1952        
(b) वर्ष 2006   
(c) वर्ष 2008   
(d) वर्ष 2009
Answer : (a) वर्ष 1952
45. प्रशांत महासागर में कौन सी धाराएं गर्म जल धाराएं नहीं है?
(a) क्यूरोशिवो       
(b) क्युराइल   
(c) पेरू धारा   
(d) ब्रिटिश कोलंबिया
Answer : (c) पेरू धारा
46. जैव विविधता के संकट के कारण बहुत सारे पशुओं की प्रजाति विलुप्त होने के कगार पर है.कौन पशु ऐसे नही है?
(a) अफ्रीकन हाथी       
(b) भारतीय चीता   
(c) गिद्ध       
(d) लाल पांडा
Answer : (d) लाल पांडा
47. विश्व में 25 हॉट स्पॉट ऑफ बायोडायवर्सिटी चिन्हित किए गए जिनमें से कितने भारतीय क्षेत्र में हैं?
(a) 1           
(b) 2           
(c) 3       
(d) 4
Answer : (b) 2
48. भारत में निम्न में से कौन वैश्विक महत्व का नहीं है?
(a) कांजीरंगा राष्ट्रीय उद्यान   
(b) केवलादेव राष्ट्रीय उद्यान   
(c) मानस राष्ट्रीय उद्यान   
(d) सरीसीका राष्ट्रीय उद्यान
Answer : (d) सरीसीका राष्ट्रीय उद्यान
49. गीनी गुणांक (ग्रामीण इलाकों) द्वारा आय-असमानता वाला दो प्रमुख राज्य है?
(a) हरियाणा, बिहार     
(b) हरियाणा, केरल   
(c) यूपी, एमपी
(d) एमपी, यूपी 
Answer : (b) हरियाणा, केरल
50. मनरेगा के तहत सबसे अधिक रोजगार किस राज्य में दिया गया (वर्ष 2009-2010) में?
(a) राजस्थान
(b) एमपी   
(c) उड़ीसा    
(d) झारखण्ड
Answer : (a) राजस्थान
51. राष्ट्रीय ग्रामीण स्वास्थ्य मिशन के तहत सबसे अधिक प्राथमिक स्वास्थ्य केन्द्र किस राज्य में चल रहा है?
(a) तमिलनाडु     
(b) यूपी
(c) एमपी        
(d) आन्ध्र प्रदेश 
Answer : (a) तमिलनाडु
52. जवाहर लाल नेहरु राष्ट्रीय सौर मिशन योजना वर्ष 2010 से प्रारंभ किया गया, जिसमें वर्ष 2022 तक 20,000 मेगावाट विद्युत उत्पादन का निर्णय लिया गया. इस मिशन को कितने चरणों में विभक्त किया गया है?
(a) दो         
(b) तीन
(c) चार
(d) पांच 
Answer : (c) चार
53. निम्नलिखित कथनों पर विचार कीजिए :
कथन-I- भारत का व्यापार घाटा 2009-10 में 109 मिलियन डालर है?
कथन-II- विदेशी मुद्रा भण्डार 280 मिलियन डालर है?
कथन-III- भारत पर वैदेशिक ऋण 295 मिलियन डालर है?
उपर्युक्त कथनों में से कौन सा/से सही है/हैं?
(a) I और II
(b) II और III   
(c) I और III
(d) I और III और II तीनों
Answer : (c) 1 और 3
54. भारत में अनुमानतः वर्ष 2010-11 के अनुसार राजस्व घाटा एवं राजकोषीय घाटा जीडीपी (GDP) का कितना प्रतिशत रहा है?
(a) 3.5, 4.8   
(b) 4.8, 3.5
(c) 3,6    
(d) 0,3   
Answer : (a) 3.5, 4.8
55. प्रत्यक्ष कर संहिता विधेयक वर्ष 2010 में क्या-2 प्रावधान नहीं है?
(a) विदेशी कम्पनियां 42.2% टैक्स देगी
(b) भारतीय कम्पनियां 33.3% टैक्स  देगी
(c) कर की सीमा 2 लाख होगी   
(d) महिलाओं को 2.5 लाख तक राहत होगी  
Answer : (d) महिलाओं को 2.5 लाख तक राहत होगी
56. उज्ज्वला योजना के अंतर्गत कौन-2 से घटक है?
      नीचे दिए गए कूट का प्रयोग कर सही उत्तर चुनिए :
(1) निवारण     (2) मुक्ति         
(3) पुनर्वास     (4) पुनर्एकीकरण     (5) पीड़ित पुनर्वापसी
कूट :
(a) 1,2,3           (b) 1,2,3,4,5   
(c) 1,2,3,4        (d) इनमें से कोई नहीं 
Answer :  (b) 1,2,3,4,5
57. वर्ष 2008-12 की अवधि के लिए बाल मृत्यु दर, एवं मात्र म्रत्यु दर में कमी करने और शिक्षा में वृद्धि, आदि  के लिए भारत सरकार कंट्री प्रोग्राम एक्सन प्लान चला रही है भारत सरकार निम्नलिखित में से किससे सहयोग ले रही है?
(a) यूएनओ (UNO)
(b) रेडक्रोंस
(c) यूनीसेफ
(d) यूएनडीपी (UNDP)
Answer : (c) यूनीसेफ
58. बाल अधिकारों की रक्षा हेतु राष्ट्रीय आयोग भारत में कब से कार्यरत है?
(a) वर्ष 2003    
(b) वर्ष 2005
(c) वर्ष 2007
(d) वर्ष 2009  
Answer : (c) वर्ष 2007
59. इंदिरा गांधी विकलांगता पेंशन योजना 2009 में प्रति बीपीएल (BPL) वाले व्यक्ति को 400 रुपए प्रति माह देने में केन्द्र सरकार का प्रतिशत क्या है?
(a) 90      
(b) 70   
(c) 50
(d) 30  
Answer : (c) 50
60. निम्न में से कौन सही सुमेलित नहीं है?
(a) बीरबल साहनी जीवाश्म संस्थान–लखनऊ
(b) रमण अनुसंधान संस्थान-?
(c) भारतीय एस्ट्रोफिजिक्स संस्थान-मुम्बई
(d) भारतीय विज्ञान प्रसार संगठन-कानपुर 
Answer : (d) भारतीय विज्ञान प्रसार संगठन-कानपुर
61. विश्व की सबसे लंबी पाइप लाइन 16 लाख किमी.का क्या नाम है?
(a) बिग इंच       
(b) ग्रेट लाइन
(c) लिटिल इंच
Answer : (a) बिग इंच
62. विश्व में लिंगानुपात कितना है?
(a) 993   
(b) 996
(c) 999
(d) 990
Answer : (a) 993
63. सूची I को सूची II से निम्न पवनों को सुमेलित कीजिए और सूचियों के नीचे दिए गए कूट का प्रयोग कर सही उत्तर चुनिए :
सूची I (पवन)         सूची II (क्षेत्र)
    A. फोन               1. सहारा
    B. चिनुक             2. फ़्रांस
    C. मिस्ट्रल           3. रॉकी पर्वत
    D. हटमटन          4. अल्पस पर्वत
कूट :
         A    B    C    D
(a)    4    3    1    2
(b)    2    3    1    4
(c)    4    3    2    1
(d)    3    4    2    1
Answer :-  (b)  2 3 1 4
64. जब वायु हल्की होकर ऊपर उठती है , एवं कपासी मेघ का निर्माण करती है, जिससे घनघोर वर्षा होती है. ऐसी वर्षा प्रायः निम्न क्षेत्र में अधिक होती है?
(a) भूमध्य रेखा   
(b) ध्रुवीय क्षेत्र
(c) उपोष्ण क्षेत्र   
(d) हिमालय क्षेत्र
Answer : (a) भूमध्य रेखा
65. टैगा (50-70 Month) जलवायु क्षेत्र का नाम किस आधार पर पड़ा है?
(a) कोणधारी वन   
(b) पर्वत
(c) जनजाति       
(d) नदी
Answer : (a) कोणधारी वन
66. विश्व का 20% खनिज तेल एवं गैस महासागर में कहां मिलती है?
(a) महाद्वीपीय मग्नतट   
(b) महाद्वीपीय ढाल
(c) महाद्वीपीय उत्थान   
(d) नितल मैदान
Answer : (a) महाद्वीपीय मग्नतट
67. हवाई उभार (Hawaiian Swell), ऐल्बाट्रास पठार एवं अटाकामा गर्त किस महासागर में पाया जाता है?
(a) प्रशांत   
(b) अटलांटिक
(c) हिंद   
(d) आर्कटिक
Answer : (a) प्रशांत
68. बाल्टिक सागर, कैरीबियन सी, हडसन की खाड़ी किस महासागर के साथ जुड़ा है?
(a) प्रशांत   
(b) अटलांटिक
(c) आर्कटिक
(d) अंटार्कटिक
Answer : (b) अटलांटिक
69. हिंद महासागर का सर्वाधिक गहरा गर्त है?
(a) टोंगा गर्त       
(b) सैंडविच
(c) सुंडा   
(d) सोकोगा गर्त
Answer : (c) सुंडा
70. महासागरों में लवणता सबसे अधिक कहां पाई जाती है?
(a) कर्क रेखा       
(b) भूमध्य रेखा
(c) ध्रूवीय रेखा
(d) मकर रेखा
Answer : (a) कर्क रेखा
71. कौन सा कथन सत्य नहीं है?
(a) लाल रंग का तरंग दैर्ध्य (wave length ) सबसे अधिक है?
(b) बैंगनी रंग का अपवर्तन ( Refraction ) सबसे अधिक है?
(c) ताप के बढ़ने के साथ अपवर्तनांक का मान कम हो जाता है?
(d) जब प्रकाश सघन माध्यम से विरल माध्यम में प्रवेश करता है. तो अभिलम्ब से निकट आता है?
Answer : (d) जब प्रकाश सघन माध्यम से विरल माध्यम में प्रवेश करता है. तो अभिलम्ब से निकट आता है
72. विद्युत की धारा को अधिक दूरी तक ले जाने के लिए क्या करें?
(a) उच्च वोल्टेज एवं निम्न करेंट   
(b) उच्च करेंट एवं निम्न वोल्टेज
(c) उच्च वोल्टेज एवं शून्य करेंट    
(d) उच्च करेंट एवं शून्य वोल्टेज
Answer : (a) उच्च वोल्टेज एवं निम्न करेंट
73. यदि यूरेनियम U235 की अर्ध आयु  (Half life) 4.5X109 वर्ष है. तो यह पूरी तरह कब क्षय हो जायेगा?
(a) 1.12X109 वर्ष
(b) 2.25X109 वर्ष   
(c) 0.56X109 वर्ष       
(d) कभी नहीं
Answer : (d) कभी नहीं
74. सभी रेडियो एक्टिव तत्व क्षय होकर किस रूप में रहते हैं?
(a) सीसा       
(b) कांच       
(c) पारा   
(d) प्लेटिनम
Answer : (a) सीसा
75. सूची I को सूची II से सुमेलित कीजिए और सूचियों के नीचे दिए गए कूट का प्रयोग कर सही उत्तर चुनिए :
         सूची I           सूची II
    A. इलेक्ट्रॉन       1. थॉमसन
    B. प्रोटॉन           2. गोल्डस्टीन
    C. न्यूट्रॉन          3. चैडविक
    D. हाइड्रोजन       4. कैम्डीस
कूट :
         A    B    C    D       
(a)    1    2    3    4
(b)    4    3    2    1
(c)    1    2    4    3
(d)    2    1    4    3
Answer : (a) 1 2 3 4
76. स्टेनलेस स्टील में आयरन के साथ कौन सा तत्व नहीं पाया जाता है?
(a) कार्बन        
(b) क्रोमियम
(c) मैगनीज   
(d) एल्युमिनियम
Answer : (d) एल्युमिनियम
77. वनस्पति तेलों से कृत्रिम घी ( डालडा ) बनाने के लिए किस उत्प्रेरक का प्रयोग करते है?
(a) निकिल   
(b) एलुमिना   
(c) लोहे का चूर्ण   
(d) प्लेटेनियम
Answer : (a) निकिल
78. कौन सा धातु विद्युत का सबसे अच्छा सुचालक है?
(a) चांदी
(b) तांबा       
(c) लोहा   
(d) एल्युमिनियम
Answer : (a) चांदी